NCLEX questions

¡Supera tus tareas y exámenes ahora con Quizwiz!

Many women seek alternative or complementary pain relief during labor to delay or avoid pharmacologic or invasive therapies. Evidence regarding the use of these modalities has been sparse, and so they remain underutilized. Published reviews of the best-known therapies identified the benefits of each modality for the woman in labor. Please match the alternative modality with the correct research finding. a. Yoga b. Massage c. Acupuncture d. Water immersion e. Aromatherapy - Less pain intensity, decreased use of analgesia, fewer instrumental births - Significantly decreased use of analgesia, shorter labor - No difference when compared with placebo - Less pain and anxiety during the first stage of labor - Reduced length of labor, increased satisfaction of pain relief

- Less pain intensity, decreased use of analgesia, fewer instrumental births: c. Acupuncture - Significantly decreased use of analgesia, shorter labor: d. Water immersion - No difference when compared with placebo: e. Aromatherapy - Less pain and anxiety during the first stage of labor: b. Massage - Reduced length of labor, increased satisfaction of pain relief: a. Yoga

At 1 minute after birth, the nurse assesses the infant and notes a heart rate of 80 beats/minute, some flexion of the extremities, a weak cry, grimacing, and a pink body with blue extremities. The nurse would calculate an Apgar score of: ________

5

Instruct the patient and partner in the use of specific relaxation techniques.

Acute pain related to contractions

Orient the patient and family to the labor and birth unit.

Anxiety related to labor and the birthing process

Ethnicity who has an IUD inserted after the first child

Chinese

Average of 10 mmHg

Duration

Puerperal sepsis

Elevated temperature at 36 hours postpartum

Dehydrating effects of labor

Elevated temperature within the first 24 hours

Group care activities as much as possible.

Fatigue related to energy expenditure during labor and birth

Generally ranging from two to five contractions per 10 minutes of labor

Frequency

Ethnicity who takes the placenta home to bury

Haitian

Excessive use of oxytocin

Hypertension

Unusually high epidural or spinal block

Hypoventilation

Ethnicity who prefers not to give babies colostrum

Korean or other South East Asian countries.

Ethnicity who eats only warm foods and hot drinks

Mexican

Ethnicity who will not eat pork or pork products

Muslim Countries

Hypovolemia resulting from hemorrhage

Rapid pulse

Commonly 45 seconds or more in the second stage of labor

Relaxation time

Remaining fairly stable throughout the first and second stages

Resting tone

Continue to provide comfort measures and minimize distractions.

Risk for impaired individual coping

Encourage frequent voiding and catheterize if necessary.

Risk for impaired urinary elimination

Peaking at 40 to 70 mmHg in the first stage of labor

Strength

Early postpartum hemorrhage is defined as a blood loss greater than: a. 500 mL in the first 24 hours after vaginal delivery. b. 750 mL in the first 24 hours after vaginal delivery. c. 1000 mL in the first 48 hours after cesarean delivery. d. 1500 mL in the first 48 hours after cesarean delivery.

a. 500 mL in the first 24 hours after vaginal delivery.

Parents have been asked by the neonatologist to provide breast milk for their newborn son, who was born prematurely at 32 weeks of gestation. The nurse who instructs them about pumping, storing, and transporting the milk needs to assess their knowledge of lactation. Which statement is valid? a. A premature infant more easily digests breast milk than formula. b. A glass of wine just before pumping will help reduce stress and anxiety. c. The mother should pump only as much as the infant can drink. d. The mother should pump every 2 to 3 hours, including during the night.

a. A premature infant more easily digests breast milk than formula.

Part of the health assessment of a newborn is observing the infants breathing pattern. A full-term newborns breathing pattern is predominantly: a. Abdominal with synchronous chest movements. b. Chest breathing with nasal flaring. c. Diaphragmatic with chest retraction. d. Deep with a regular rhythm.

a. Abdominal with synchronous chest movements.

A new mother states that her infant must be cold because the babys hands and feet are blue. The nurse explains that this is a common and temporary condition called: a. Acrocyanosis. b. Erythema neonatorum. c. Harlequin color. d. Vernix caseosa.

a. Acrocyanosis.

Possible alternative and complementary therapies for postpartum depression (PPD) for breastfeeding mothers include (Select all that apply): a. Acupressure. b. Aromatherapy. c. St. Johns wort. d. Wine Consumption. e. Yoga.

a. Acupressure. b. Aromatherapy. e. Yoga.

The abuse of which of the following substances during pregnancy is the leading cause of cognitive impairment in the United States? a. Alcohol b. Tobacco c. Marijuana d. Heroin

a. Alcohol

To care for a laboring woman adequately, the nurse understands that the __________ stage of labor varies the most in length? a. First b. Second c. Third d. Fourth

a. First

A pregnant woman is in her third trimester. She asks the nurse to explain how she can tell true labor from false labor. The nurse would explain that true labor contractions: a. Increase with activity such as ambulation. b. Decrease with activity. c. Are always accompanied by the rupture of the bag of waters. d. Alternate between a regular and an irregular pattern.

a. Increase with activity such as ambulation.

A woman gave birth to an infant boy 10 hours ago. Where would the nurse expect to locate this womans fundus? a. One centimeter above the umbilicus b. Two centimeters below the umbilicus c. Midway between the umbilicus and the symphysis pubis d. Nonpalpable abdominally

a. One centimeter above the umbilicus

A woman in preterm labor at 30 weeks of gestation receives two 12-mg doses of betamethasone intramuscularly. The purpose of this pharmacologic treatment is to: a. Stimulate fetal surfactant production. b. Reduce maternal and fetal tachycardia associated with ritodrine administration. c. Suppress uterine contractions. d. Maintain adequate maternal respiratory effort and ventilation during magnesium sulfate therapy.

a. Stimulate fetal surfactant production.

Nurses play a critical role in educating parents regarding measures to prevent infant abduction. Which instructions contribute to infant safety and security? a. The mother should check the photo ID of any person who comes to her room. b. The baby should be carried in the parents arms from the room to the nursery. c. Because of infant security systems, the baby can be left unattended in the patients room. d. Parents should use caution when posting photos of their infant on the Internet. e. The mom should request that a second staff member verify the identity of any questionable person.

a. The mother should check the photo ID of any person who comes to her room. d. Parents should use caution when posting photos of their infant on the Internet. e. The mom should request that a second staff member verify the identity of any questionable person.

Which maternal condition is considered a contraindication for the application of internal monitoring devices? a. Unruptured membranes b. Cervix dilated to 4 cm c. External monitors in current use d. Fetus with a known heart defect

a. Unruptured membranes

Examples of appropriate techniques to wake a sleepy infant for breastfeeding include (Select all that apply): a. Unwrapping the infant. b. Changing the diaper. c. Talking to the infant. d. Slapping the infants hands and feet. e. Applying a cold towel to the infants abdomen.

a. Unwrapping the infant. b. Changing the diaper. c. Talking to the infant.

A client is warm and asks for a fan in her room for her comfort. The nurse enters the room to assess the mother and her infant and finds the infant unwrapped in his crib with the fan blowing over him on high. The nurse instructs the mother that the fan should not be directed toward the newborn and the newborn should be wrapped in a blanket. The mother asks why. The nurses best response is: a. Your baby may lose heat by convection, which means that he will lose heat from his body to the cooler ambient air. You should keep him wrapped and prevent cool air from blowing on him. b. Your baby may lose heat by conduction, which means that he will lose heat from his body to the cooler ambient air. You should keep him wrapped and prevent cool air from blowing on him. c. Your baby may lose heat by evaporation, which means that he will lose heat from his body to the cooler ambient air. You should keep him wrapped and prevent cool air from blowing on him. d. Your baby will get cold stressed easily and needs to be bundled up at all times.

a. Your baby may lose heat by convection, which means that he will lose heat from his body to the cooler ambient air. You should keep him wrapped and prevent cool air from blowing on him.

During a prenatal examination, the woman reports having two cats at home. The nurse informs her that she should not be cleaning the litter box while she is pregnant. When the woman asks why, the nurses best response would be: a. Your cats could be carrying toxoplasmosis. This is a zoonotic parasite that can infect you and have severe effects on your unborn child. b. You and your baby can be exposed to the human immunodeficiency virus (HIV) in your cats feces. c. Its just gross. You should make your husband clean the litter boxes. d. Cat feces are known to carry Escherichia coli, which can cause a severe infection in both you and your baby.

a. Your cats could be carrying toxoplasmosis. This is a zoonotic parasite that can infect you and have severe effects on your unborn child.

The nurse is caring for a client whose labor is being augmented with oxytocin. He or she recognizes that the oxytocin should be discontinued immediately if there is evidence of: a. Uterine contractions occurring every 8 to 10 minutes. b. A fetal heart rate (FHR) of 180 with absence of variability. c. The clients needing to void. d. Rupture of the clients amniotic membranes.

b. A fetal heart rate (FHR) of 180 with absence of variability.

During labor a fetus with an average heart rate of 135 beats/min over a 10-minute period would be considered to have: a. Bradycardia. b. A normal baseline heart rate. c. Tachycardia. d. Hypoxia.

b. A normal baseline heart rate.

A woman is diagnosed with having a stillborn. At first, she appears stunned by the news, cries a little, and then asks you to call her mother. The phase of bereavement the woman is experiencing is called: a. Anticipatory grief. b. Acute distress. c. Intense grief. d. Reorganization.

b. Acute distress.

With regard to the postpartum uterus, nurses should be aware that: a. At the end of the third stage of labor it weighs approximately 500 g. b. After 2 weeks postpartum it should not be palpable abdominally. c. After 2 weeks postpartum it weighs 100 g. d. It returns to its original (prepregnancy) size by 6 weeks postpartum.

b. After 2 weeks postpartum it should not be palpable abdominally.

As relates to fetal positioning during labor, nurses should be aware that: a. Position is a measure of the degree of descent of the presenting part of the fetus through the birth canal. b. Birth is imminent when the presenting part is at +4 to +5 cm below the spine. c. The largest transverse diameter of the presenting part is the suboccipitobregmatic diameter. d. Engagement is the term used to describe the beginning of labor.

b. Birth is imminent when the presenting part is at +4 to +5 cm below the spine.

To teach patients about the process of labor adequately, the nurse knows that which event is the best indicator of true labor? a. Bloody show b. Cervical dilation and effacement c. Fetal descent into the pelvic inlet d. Uterine contractions every 7 minutes

b. Cervical dilation and effacement

Many first-time parents do not plan on their parents help immediately after the newborn arrives. What statement by the nurse is the most appropriate when counseling new parents about the involvement of grandparents? a. You should tell your parents to leave you alone. b. Grandparents can help you with parenting skills and also help preserve family traditions. c. Grandparent involvement can be very disruptive to the family. d. They are getting old. You should let them be involved while they can.

b. Grandparents can help you with parenting skills and also help preserve family traditions.

To assist the woman after delivery of the infant, the nurse knows that the blood patch is used after spinal anesthesia to relieve: a. Hypotension. b. Headache. c. Neonatal respiratory depression. d. Loss of movement.

b. Headache.

The most common cause of pathologic hyperbilirubinemia is: a. Hepatic disease. b. Hemolytic disorders in the newborn. c. Postmaturity. d. Congenital heart defect.

b. Hemolytic disorders in the newborn.

If an opioid antagonist is administered to a laboring woman, she should be told that: a. Her pain will decrease. b. Her pain will return. c. She will feel less anxious. d. She will no longer feel the urge to push.

b. Her pain will return.

Cardiovascular changes that cause the foramen ovale to close at birth are a direct result of: a. Increased pressure in the right atrium. b. Increased pressure in the left atrium. c. Decreased blood flow to the left ventricle. d. Changes in the hepatic blood flow.

b. Increased pressure in the left atrium.

A woman who is 39 weeks pregnant expresses fear about her impending labor and how she will manage. The nurses best response is: a. Don't worry about it. You'll do fine. b. Its normal to be anxious about labor. Lets discuss what makes you afraid. c. Labor is scary to think about, but the actual experience isn't. d. You can have an epidural. You wont feel anything.

b. Its normal to be anxious about labor. Lets discuss what makes you afraid.

Which instructions should be included in the discharge teaching plan to assist the patient in recognizing early signs of complications? a. Palpate the fundus daily to ensure that it is soft. b. Notify the physician of any increase in the amount of lochia or a return to bright red bleeding. c. Report any decrease in the amount of brownish red lochia. d. The passage of clots as large as an orange can be expected.

b. Notify the physician of any increase in the amount of lochia or a return to bright red bleeding.

What is an expected characteristic of amniotic fluid? a. Deep yellow color b. Pale, straw color with small white particles c. Acidic result on a Nitrazine test d. Absence of ferning

b. Pale, straw color with small white particles

A new father wants to know what medication was put into his infants eyes and why it is needed. The nurse explains to the father that the purpose of the Ilotycin ophthalmic ointment is to: a. Destroy an infectious exudate caused by Staphylococcus that could make the infant blind. b. Prevent gonorrheal and chlamydial infection of the infants eyes potentially acquired from the birth canal. c. Prevent potentially harmful exudate from invading the tear ducts of the infants eyes, leading to dry eyes. d. Prevent the infants eyelids from sticking together and help the infant see.

b. Prevent gonorrheal and chlamydial infection of the infants eyes potentially acquired from the birth canal.

The hormone necessary for milk production is: a. Estrogen. b. Prolactin. c. Progesterone. d. Lactogen.

b. Prolactin.

After change-of-shift report the nurse assumes care of a multiparous client in labor. The woman is complaining of pain that radiates to her abdominal wall, lower back, and buttocks and down her thighs. Before implementing a plan of care, the nurse should understand that this type of pain is: a. Visceral. b. Referred. c. Somatic. d. Afterpain.

b. Referred.

After an emergency birth, the nurse encourages the woman to breastfeed her newborn. The primary purpose of this activity is to: a. Facilitate maternal-newborn interaction. b. Stimulate the uterus to contract. c. Prevent neonatal hypoglycemia. d. Initiate the lactation cycle.

b. Stimulate the uterus to contract.

The perinatal nurse assisting with establishing lactation is aware that acute mastitis can be minimized by: a. Washing the nipples and breasts with mild soap and water once a day. b. Using proper breastfeeding techniques. c. Wearing a nipple shield for the first few days of breastfeeding. d. Wearing a supportive bra 24 hours a day.

b. Using proper breastfeeding techniques.

The nurse should be aware that a pessary would be most effective in the treatment of what disorder? a. Cystocele b. Uterine prolapse c. Rectocele d. Stress urinary incontinence

b. Uterine prolapse

To provide safe care for the woman, the nurse understands that which condition is a contraindication for an amniotomy? a. Dilation less than 3 cm b. Cephalic presentation c. -2 station d. Right occiput posterior position

c. -2 station

While assessing the newborn, the nurse should be aware that the average expected apical pulse range of a full-term, quiet, alert newborn is: a. 80 to 100 beats/min. b. 100 to 120 beats/min. c. 120 to 160 beats/min. d. 150 to 180 beats/min.

c. 120 to 160 beats/min.

Changes in blood volume after childbirth depend on several factors such as blood loss during childbirth and the amount of extravascular water (physiologic edema) mobilized and excreted. A postpartum nurse anticipates blood loss of (Select all that apply): a. 100 mL b. 250 mL or less c. 300 to 500 mL d. 500 to 1000 mL e. 1500 mL or greater

c. 300 to 500 mL d. 500 to 1000 mL

Under the Newborns and Mothers Health Protection Act, all health plans are required to allow new mothers and newborns to remain in the hospital for a minimum of _____ hours after a normal vaginal birth and for _____ hours after a cesarean birth. a. 24, 73 b. 24, 96 c. 48, 96 d. 48, 120

c. 48, 96

At 1 minute after birth, the nurse assesses the newborn to assign an Apgar score. The apical heart rate is 110 bpm, and the infant is crying vigorously with the limbs flexed. The infants trunk is pink, but the hands and feet are blue. What is the correct Apgar score for this infant? a. 7 b. 8 c. 9 d. 10

c. 9

How many kilocalories per kilogram (kcal/kg) of body weight does a breastfed term infant require each day? a. 50 to 65 b. 75 to 90 c. 95 to 110 d. 150 to 200

c. 95 to 110

When assessing a woman in labor, the nurse is aware that the relationship of the fetal body parts to one another is called fetal: a. Lie. b. Presentation. c. Attitude. d. Position.

c. Attitude.

A mother expresses fear about changing her infants diaper after he is circumcised. What does the woman need to be taught to take care of the infant when she gets home? a. Cleanse the penis with prepackaged diaper wipes every 3 to 4 hours. b. Apply constant, firm pressure by squeezing the penis with the fingers for at least 5 minutes if bleeding occurs. c. Cleanse the penis gently with water and put petroleum jelly around the glans after each diaper change. d. Wash off the yellow exudate that forms on the glans at least once every day to prevent infection.

c. Cleanse the penis gently with water and put petroleum jelly around the glans after each diaper change.

Excessive blood loss after childbirth can have several causes; the most common is: a. Vaginal or vulvar hematomas. b. Unrepaired lacerations of the vagina or cervix. c. Failure of the uterine muscle to contract firmly. d. Retained placental fragments.

c. Failure of the uterine muscle to contract firmly.

A recently delivered mother and her baby are at the clinic for a 6-week postpartum checkup. The nurse should be concerned that psychosocial outcomes are not being met if the woman: a. Discusses her labor and birth experience excessively. b. Believes that her baby is more attractive and clever than any others. c. Has not given the baby a name. d. Has a partner or family members who react very positively about the baby.

c. Has not given the baby a name.

While examining a newborn, the nurse notes uneven skin folds on the buttocks and a click when performing the Ortolani maneuver. The nurse recognizes these findings as a sign that the newborn probably has: a. Polydactyly. b. Clubfoot. c. Hip dysplasia. d. Webbing

c. Hip dysplasia.

To provide adequate postpartum care, the nurse should be aware that postpartum depression (PPD) without psychotic features: a. Means that the woman is experiencing the baby blues. In addition she has a visit with a counselor or psychologist. b. Is more common among older, Caucasian women because they have higher expectations. c. Is distinguished by irritability, severe anxiety, and panic attacks. d. Will disappear on its own without outside help.

c. Is distinguished by irritability, severe anxiety, and panic attacks.

With regard to afterbirth pains, nurses should be aware that these pains are: a. Caused by mild, continuous contractions for the duration of the postpartum period. b. More common in first-time mothers. c. More noticeable in births in which the uterus was overdistended. d. Alleviated somewhat when the mother breastfeeds.

c. More noticeable in births in which the uterus was overdistended.

Anxiety disorders are the most common mental disorders that affect women. While providing care to the maternity patient, the nurse should be aware that one of these disorders is likely to be triggered by the process of labor and birth. This disorder is: a. Phobias. b. Panic disorder. c. Post-traumatic stress disorder (PTSD). d. Obsessive-compulsive disorder (OCD).

c. Post-traumatic stress disorder (PTSD).

A woman has requested an epidural for her pain. She is 5 cm dilated and 100% effaced. The baby is in a vertex position and is engaged. The nurse increases the woman's intravenous fluid for a preprocedural bolus. She reviews her laboratory values and notes that the woman's hemoglobin is 12 g/dL, hematocrit is 38%, platelets are 67,000, and white blood cells (WBCs) are 12,000/mm3. Which factor would contraindicate an epidural for the woman? a. She is too far dilated. b. She is anemic. c. She has thrombocytopenia. d. She is septic.

c. She has thrombocytopenia.

It is paramount for the obstetric nurse to understand the regulatory procedures and criteria for admitting a woman to the hospital labor unit. Which guideline is an important legal requirement of maternity care? a. The patient is not considered to be in true labor (according to the Emergency Medical Treatment and Active Labor Act [EMTALA]) until a qualified health care provider says she is. b. The woman can have only her male partner or predesignated doula with her at assessment. c. The patients weight gain is calculated to determine whether she is at greater risk for cephalopelvic disproportion (CPD) and cesarean birth. d. The nurse may exchange information about the patient with family members.

c. The patients weight gain is calculated to determine whether she is at greater risk for cephalopelvic disproportion (CPD) and cesarean birth.

In assisting the breastfeeding mother position the baby, nurses should keep in mind that: a. The cradle position usually is preferred by mothers who had a cesarean birth. b. Women with perineal pain and swelling prefer the modified cradle position. c. Whatever the position used, the infant is belly to belly with the mother. d. While supporting the head, the mother should push gently on the occiput.

c. Whatever the position used, the infant is belly to belly with the mother.

Which options for saying goodbye would the nurse want to discuss with a woman who is diagnosed with having a stillborn girl? a. The nurse shouldn't discuss any options at this time; there is plenty of time after the baby is born. b. Would you like a picture taken of your baby after birth? c. When your baby is born, would you like to see and hold her? d. What funeral home do you want notified after the baby is born?

c. When your baby is born, would you like to see and hold her?

A postpartum woman overhears the nurse tell the obstetrics clinician that she has a positive Homan's sign and asks what it means. The nurses best response is: a. You have pitting edema in your ankles. b. You have deep tendon reflexes rated 2+. c. You have calf pain when the nurse flexes your foot. d. You have a fleshy odor to your vaginal drainage.

c. You have calf pain when the nurse flexes your foot.

An infant is being discharged from the neonatal intensive care unit after 70 days of hospitalization. The infant was born at 30 weeks of gestation with several conditions associated with prematurity, including respiratory distress syndrome, mild bronchopulmonary dysplasia, and retinopathy of prematurity requiring surgical treatment. During discharge teaching the infants mother asks the nurse whether her baby will meet developmental milestones on time, as did her son who was born at term. The nurses most appropriate response is: a. Your baby will develop exactly like your first child did. b. Your baby does not appear to have any problems at the present time. c. Your baby will need to be corrected for prematurity. Your baby is currently 40 weeks of postconceptional age and can be expected to be doing what a 40-week-old infant would be doing. d. Your baby will need to be followed very closely.

c. Your baby will need to be corrected for prematurity. Your baby is currently 40 weeks of postconceptional age and can be expected to be doing what a 40-week-old infant would be doing.

A new mother asks the nurse when the soft spot on her sons head will go away. The nurses answer is based on the knowledge that the anterior fontanel closes after birth by _____ months. a. 2 b. 8 c. 12 d. 18

d. 18

Which collection of risk factors most likely would result in damaging lacerations (including episiotomies)? a. A dark-skinned woman who has had more than one pregnancy, who is going through prolonged second-stage labor, and who is attended by a midwife b. A reddish-haired mother of two who is going through a breech birth c. A dark-skinned, first-time mother who is going through a long labor d. A first-time mother with reddish hair whose rapid labor was overseen by an obstetrician

d. A first-time mother with reddish hair whose rapid labor was overseen by an obstetrician

The nurse practicing in a labor setting knows that the woman most at risk for uterine rupture is: a. A gravida 3 who has had two low-segment transverse cesarean births. b. A gravida 2 who had a low-segment vertical incision for delivery of a 10-pound infant. c. A gravida 5 who had two vaginal births and two cesarean births. d. A gravida 4 who has had all cesarean births.

d. A gravida 4 who has had all cesarean births.

A 25-year-old gravida 1 para 1 who had an emergency cesarean birth 3 days ago is scheduled for discharge. As you prepare her for discharge, she begins to cry. Your initial action should be to a. Assess her for pain. b. Point out how lucky she is to have a healthy baby. c. Explain that she is experiencing postpartum blues. d. Allow her time to express her feelings.

d. Allow her time to express her feelings.

Which statement describing physiologic jaundice is incorrect? a. Neonatal jaundice is common, but kernicterus is rare. b. The appearance of jaundice during the first 24 hours or beyond day 7 indicates a pathologic process. c. Because jaundice may not appear before discharge, parents need instruction on how to assess it and when to call for medical help. d. Breastfed babies have a lower incidence of jaundice.

d. Breastfed babies have a lower incidence of jaundice.

Which statement concerning the benefits or limitations of breastfeeding is inaccurate? a. Breast milk changes over time to meet changing needs as infants grow. b. Long-term studies have shown that the benefits of breast milk continue after the infant is weaned. c. Breast milk/breastfeeding may enhance cognitive development. d. Breastfeeding increases the risk of childhood obesity.

d. Breastfeeding increases the risk of childhood obesity.

The most common cause of decreased variability in the fetal heart rate (FHR) that lasts 30 minutes or less is: a. Altered cerebral blood flow. b. Fetal hypoxemia. c. Umbilical cord compression. d. Fetal sleep cycles.

d. Fetal sleep cycles.

The nurse assessing a newborn knows that the most critical physiologic change required of the newborn is: a. Closure of fetal shunts in the circulatory system. b. Full function of the immune defense system at birth. c. Maintenance of a stable temperature. d. Initiation and maintenance of respirations.

d. Initiation and maintenance of respirations.

On examining a woman who gave birth 5 hours ago, the nurse finds that the woman has completely saturated a perineal pad within 15 minutes. The nurses first action is to: a. Begin an intravenous (IV) infusion of Ringers lactate solution. b. Assess the woman's vital signs. c. Call the woman's primary health care provider. d. Massage the woman's fundus.

d. Massage the woman's fundus

Which condition is a transient, self-limiting mood disorder that affects new mothers after childbirth? a. Postpartum depression b. Postpartum psychosis c. Postpartum bipolar disorder d. Postpartum blues

d. Postpartum blues

The factors that affect the process of labor and birth, known commonly as the five Ps, include all except: a. Passenger. b. Passageway. c. Powers. d. Pressure.

d. Pressure.

During life in utero, oxygenation of the fetus occurs through transplacental gas exchange. When birth occurs, four factors combine to stimulate the respiratory center in the medulla. The initiation of respiration then follows. Which is not one of these essential factors? a. Chemical b. Mechanical c. Thermal d. Psychologic

d. Psychologic

When teaching parents about mandatory newborn screening, it is important for the nurse to explain that the main purpose is to: a. Keep the state records updated. b. Allow accurate statistical information. c. Document the number of births. d. Recognize and treat newborn disorders early.

d. Recognize and treat newborn disorders early.

With regard to postpartum ovarian function, nurses should be aware that: a. Almost 75% of women who do not breastfeed resume menstruating within a month after birth. b. Ovulation occurs slightly earlier for breastfeeding women. c. Because of menstruation/ovulation schedules, contraception considerations can be postponed until after the puerperium. d. The first menstrual flow after childbirth usually is heavier than normal.

d. The first menstrual flow after childbirth usually is heavier than normal.

When using intermittent auscultation (IA) for fetal heart rate, nurses should be aware that: a. They can be expected to cover only two or three clients when IA is the primary method of fetal assessment. b. The best course is to use the descriptive terms associated with electronic fetal monitoring (EFM) when documenting results. c. If the heartbeat cannot be found immediately, a shift must be made to EFM. d. Ultrasound can be used to find the fetal heartbeat and reassure the mother if initial difficulty was a factor.

d. Ultrasound can be used to find the fetal heartbeat and reassure the mother if initial difficulty was a factor.

What infant response to cool environmental conditions is either not effective or not available to them? a. Constriction of peripheral blood vessels b. Metabolism of brown fat c. Increased respiratory rates d. Unflexing from the normal position

d. Unflexing from the normal position

When caring for a postpartum woman experiencing hemorrhagic shock, the nurse recognizes that the most objective and least invasive assessment of adequate organ perfusion and oxygenation is: a. Absence of cyanosis in the buccal mucosa. b. Cool, dry skin. c. Diminished restlessness. d. Urinary output of at least 30 mL/hr.

d. Urinary output of at least 30 mL/hr.

A primiparous woman is delighted with her newborn son and wants to begin breastfeeding as soon as possible. The nurse can facilitate the infants correct latch-on by helping the woman hold the infant: a. With his arms folded together over his chest. b. Curled up in a fetal position. c. With his head cupped in her hand. d. With his head and body in alignment.

d. With his head and body in alignment.

The nurse caring for the laboring woman should understand that early decelerations are caused by: a. Altered fetal cerebral blood flow. b. Umbilical cord compression. c. Uteroplacental insufficiency. d. Spontaneous rupture of membranes.

a. Altered fetal cerebral blood flow.

Many common drugs of abuse cause significant physiologic and behavioral problems in infants who are breastfed by mothers currently using (Select all that apply): a. Amphetamine. b. Heroin. c. Nicotine. d. PCP. e. Morphine.

a. Amphetamine. b. Heroin. c. Nicotine. d. PCP.

A normal uterine activity pattern in labor is characterized by: a. Contractions every 2 to 5 minutes. b. Contractions lasting about 2 minutes. c. Contractions about 1 minute apart. d. A contraction intensity of about 1000 mm Hg with relaxation at 50 mm Hg.

a. Contractions every 2 to 5 minutes.

The vaginal examination is an essential component of labor assessment. It reveals whether the patient is in true labor and enables the examiner to determine whether membranes have ruptured. This examination is often stressful and uncomfortable for the patient and should be performed only when indicated. Please match the correct step number, from 1 to 7, with each component of a vaginal examination of the laboring woman. a. After obtaining permission, gently insert the index and middle fingers into the vagina. b. Explain findings to the patient. c. Position the woman to prevent supine hypotension. d. Use sterile gloves and soluble gel for lubrication. e. Document findings and report to the provider. f. Cleanse the perineum and vulva if necessary. g. Determine dilation, presenting part, status of membranes, and characteristics of amniotic fluid.

1 - d. Use sterile gloves and soluble gel for lubrication. 2 - c. Position the woman to prevent supie hypotension. 3 - f. Cleanse the perineum and vulva if necessary. 4 - a. After obtaining permission, gently insert the index and middle fingers into the vagina. 5 - g. Determine dilation, presenting part, status of membranes, and characteristics of amniotic fluid. 6 - b. Explain findings to the patient. 7 - e. Document findings and report to the provider.

For vaginal birth to be successful, the fetus must adapt to the birth canal during the descent. The turns and other adjustments necessary in the human birth process are termed the mechanism of labor. Please list the seven cardinal movements in the mechanism of labor in the correct order. a. Flexion b. Internal rotation c. External rotation d. Expulsion e. Engagement f. Descent g. Extension

1 - e. Engagement 2 - f. Descent 3 - a. Flexion 4 - b. Internal rotation 5 - g. Extension 6 - c. External rotation 7 - d. Expulsion

The process in which bilirubin is changed from a fat-soluble product to a water-soluble product is known as: a. Enterohepatic circuit. b. Conjugation of bilirubin. c. Unconjugation of bilirubin. d. Albumin binding.

b. Conjugation of bilirubin.

What are modes of heat loss in the newborn (Select all that apply)? a. Perspiration b. Convection c. Radiation d. Conduction e. Urination

b. Convection c. Radiation d. Conduction

When a woman is diagnosed with postpartum depression (PPD) with psychotic features, one of the main concerns is that she may: a. Have outbursts of anger. b. Neglect her hygiene. c. Harm her infant. d. Lose interest in her husband.

c. Harm her infant.

A pregnant womans amniotic membranes rupture. Prolapsed umbilical cord is suspected. What intervention would be the top priority? a. Placing the woman in the knee-chest position b. Covering the cord in sterile gauze soaked in saline c. Preparing the woman for a cesarean birth d. Starting oxygen by face mask

a. Placing the woman in the knee-chest position

The priority nursing care associated with an oxytocin (Pitocin) infusion is: a. Measuring urinary output. b. Increasing infusion rate every 30 minutes. c. Monitoring uterine response. d. Evaluating cervical dilation.

c. Monitoring uterine response.

Which hormone remains elevated in the immediate postpartum period of the breastfeeding woman? a. Estrogen b. Progesterone c. Prolactin d. Human placental lactogen

c. Prolactin

The nurse can help a father in his transition to parenthood by: a. Pointing out that the infant turned at the sound of his voice. b. Encouraging him to go home to get some sleep. c. Telling him to tape the infants diaper a different way. d. Suggesting that he let the infant sleep in the bassinet.

a. Pointing out that the infant turned at the sound of his voice.

In evaluating the effectiveness of oxytocin induction, the nurse would expect: a. Contractions lasting 40 to 90 seconds, 2 to 3 minutes apart. b. The intensity of contractions to be at least 110 to 130 mm Hg. c. Labor to progress at least 2 cm/hr dilation. d. At least 30 mU/min of oxytocin will be needed to achieve cervical dilation.

a. Contractions lasting 40 to 90 seconds, 2 to 3 minutes apart.

Risk factors associated with necrotizing enterocolitis (NEC) include (Select all that apply): a. Polycythemia. b. Anemia. c. Congenital heart disease. d. Bronchopulmonary dysphasia. e. Retinopathy.

a. Polycythemia. b. Anemia. c. Congenital heart disease.

When preparing to administer a hepatitis B vaccine to a newborn, the nurse should: a. Obtain a syringe with a 25-gauge, 5/8-inch needle. b. Confirm that the newborns mother has been infected with the hepatitis B virus. c. Assess the dorsogluteal muscle as the preferred site for injection. d. Confirm that the newborn is at least 24 hours old.

a. Obtain a syringe with a 25-gauge, 5/8-inch needle.

The nurse knows that proper placement of the tocotransducer for electronic fetal monitoring is located: a. Over the uterine fundus. b. On the fetal scalp. c. Inside the uterus. d. Over the mothers lower abdomen.

a. Over the uterine fundus.

Which patient status is an acceptable indication for serial oxytocin induction of labor? a. Past 42 weeks gestation b. Multiple fetuses c. Polyhydramnios d. History of long labors

a. Past 42 weeks gestation

38. Medications used to manage postpartum hemorrhage (PPH) include (Select all that apply): a. Pitocin. b. Methergine. c. Terbutaline. d. Hemabate. e. Magnesium sulfate.

a. Pitocin. b. Methergine. d. Hemabate.

According to Becks studies, what risk factor for postpartum depression is likely to have the greatest effect on the woman's condition? a. Prenatal depression b. Single-mother status c. Low socioeconomic status d. Unplanned or unwanted pregnancy

a. Prenatal depression

A pregnant woman presents in labor at term, having had no prenatal care. After birth her infant is noted to be small for gestational age with small eyes and a thin upper lip. The infant also is microcephalic. On the basis of her infants physical findings, this woman should be questioned about her use of which substance during pregnancy? a. Alcohol b. Cocaine c. Heroin d. Marijuana

a. Alcohol

A 3.8-kg infant was delivered vaginally at 39 weeks after a 30-minute second stage. There was a nuchal cord. After birth the infant is noted to have petechiae over the face and upper back. Information given to the infants parents should be based on the knowledge that petechiae: a. Are benign if they disappear within 48 hours of birth. b. Result from increased blood volume. c. Should always be further investigated. d. Usually occur with forceps delivery.

a. Are benign if they disappear within 48 hours of birth.

The nurse providing care for the laboring woman comprehends that accelerations with fetal movement: a. Are reassuring. b. Are caused by umbilical cord compression. c. Warrant close observation. d. Are caused by uteroplacental insufficiency.

a. Are reassuring.

A laboring woman is lying in the supine position. The most appropriate nursing action at this time is to: a. Ask her to turn to one side. b. Elevate her feet and legs. c. Take her blood pressure. d. Determine whether fetal tachycardia is present.

a. Ask her to turn to one side.

Immediately after the forceps-assisted birth of an infant, the nurse should: a. Assess the infant for signs of trauma. b. Give the infant prophylactic antibiotics. c. Apply a cold pack to the infants scalp. d. Measure the circumference of the infants head.

a. Assess the infant for signs of trauma.

Discharge instruction, or teaching the woman what she needs to know to care for herself and her newborn, officially begins: a. At the time of admission to the nurses unit. b. When the infant is presented to the mother at birth. c. During the first visit with the physician in the unit. d. When the take-home information packet is given to the couple.

a. At the time of admission to the nurses unit.

New parents express concern that, because of the mothers emergency cesarean birth under general anesthesia, they did not have the opportunity to hold and bond with their daughter immediately after her birth. The nurses response should convey to the parents that: a. Attachment, or bonding, is a process that occurs over time and does not require early contact. b. The time immediately after birth is a critical period for people. c. Early contact is essential for optimum parent-infant relationships. d. They should just be happy that the infant is healthy.

a. Attachment, or bonding, is a process that occurs over time and does not require early contact.

The self-destruction of excess hypertrophied tissue in the uterus is called: a. Autolysis. b. Subinvolution. c. Afterpain. d. Diastasis.

a. Autolysis.

Which newborn reflex is elicited by stroking the lateral sole of the infants foot from the heel to the ball of the foot? a. Babinski b. Tonic neck c. Stepping d. Plantar grasp

a. Babinski

Which occurrence is associated with cervical dilation and effacement? a. Bloody show b. False labor c. Lightening d. Bladder distention

a. Bloody show

A nurse is discussing the signs and symptoms of mastitis with a mother who is breastfeeding. What signs and symptoms should the nurse include in her discussion (Select all that apply)? a. Breast tenderness b. Warmth in the breast c. An area of redness on the breast often resembling the shape of a pie wedge d. A small white blister on the tip of the nipple e. Fever and flulike symptoms

a. Breast tenderness b. Warmth in the breast c. An area of redness on the breast often resembling the shape of a pie wedge e. Fever and flulike symptoms

Knowing that the condition of the new mothers breasts will be affected by whether she is breastfeeding, nurses should be able to tell their clients all the following statements except: a. Breast tenderness is likely to persist for about a week after the start of lactation. b. As lactation is established, a mass may form that can be distinguished from cancer by its position shift from day to day. c. In nonlactating mothers colostrum is present for the first few days after childbirth. d. If suckling is never begun (or is discontinued), lactation ceases within a few days to a week.

a. Breast tenderness is likely to persist for about a week after the start of lactation.

The nurse providing couplet care should understand that nipple confusion results when: a. Breastfeeding babies receive supplementary bottle feedings. b. The baby is weaned too abruptly. c. Pacifiers are used before breastfeeding is established. d. Twins are breastfed together.

a. Breastfeeding babies receive supplementary bottle feedings.

With regard to breathing techniques during labor, maternity nurses should understand that: a. Breathing techniques in the first stage of labor are designed to increase the size of the abdominal cavity to reduce friction. b. By the time labor has begun, it is too late for instruction in breathing and relaxation. c. Controlled breathing techniques are most difficult near the end of the second stage of labor. d. The patterned-paced breathing technique can help prevent hyperventilation.

a. Breathing techniques in the first stage of labor are designed to increase the size of the abdominal cavity to reduce friction.

Which presentation is described accurately in terms of both presenting part and frequency of occurrence? a. Cephalic: occiput; at least 95% b. Breech: sacrum; 10% to 15% c. Shoulder: scapula; 10% to 15% d. Cephalic: cranial; 80% to 85%

a. Cephalic: occiput; at least 95%

The nurse caring for a laboring woman is aware that maternal cardiac output can be increased by: a. Change in position. b. Oxytocin administration. c. Regional anesthesia. d. Intravenous analgesic.

a. Change in position.

While evaluating an external monitor tracing of a woman in active labor, the nurse notes that the fetal heart rate (FHR) for five sequential contractions begins to decelerate late in the contraction, with the nadir of the decelerations occurring after the peak of the contraction. The nurses first priority is to: a. Change the woman's position. b. Notify the care provider. c. Assist with amnioinfusion. d. Insert a scalp electrode.

a. Change the woman's position.

A new mother asks whether she should feed her newborn colostrum, because it is not real milk. The nurses most appropriate answer is: a. Colostrum is high in antibodies, protein, vitamins, and minerals. b. Colostrum is lower in calories than milk and should be supplemented by formula. c. Giving colostrum is important in helping the mother learn how to breastfeed before she goes home. d. Colostrum is unnecessary for newborns.

a. Colostrum is high in antibodies, protein, vitamins, and minerals.

Perinatal nurses are legally responsible for: a. Correctly interpreting fetal heart rate (FHR) patterns, initiating appropriate nursing interventions, and documenting the outcomes. b. Greeting the client on arrival, assessing her, and starting an intravenous line. c. Applying the external fetal monitor and notifying the care provider. d. Making sure that the woman is comfortable.

a. Correctly interpreting fetal heart rate (FHR) patterns, initiating appropriate nursing interventions, and documenting the outcomes.

A woman is experiencing back labor and complains of intense pain in her lower back. An effective relief measure would be to use: a. Counterpressure against the sacrum. b. Pant-blow (breaths and puffs) breathing techniques. c. Effleurage. d. Conscious relaxation or guided imagery.

a. Counterpressure against the sacrum.

While developing an intrapartum care plan for the patient in early labor, it is important that the nurse recognize that psychosocial factors may influence a womans experience of pain. These include (Select all that apply): a. Culture. b. Anxiety and fear. c. Previous experiences with pain. d. Intervention of caregivers. e. Support systems.

a. Culture b. Anxiety and fear. c. Previous experiences with pain. e. Support systems

When assessing a woman in the first stage of labor, the nurse recognizes that the most conclusive sign that uterine contractions are effective would be: a. Dilation of the cervix. b. Descent of the fetus. c. Rupture of the amniotic membranes. d. Increase in bloody show.

a. Dilation of the cervix.

Nurses can prevent evaporative heat loss in the newborn by: a. Drying the baby after birth and wrapping the baby in a dry blanket. b. Keeping the baby out of drafts and away from air conditioners. c. Placing the baby away from the outside wall and the windows. d. Warming the stethoscope and the nurses hands before touching the baby.

a. Drying the baby after birth and wrapping the baby in a dry blanket.

Which deceleration of the fetal heart rate would not require the nurse to change the maternal position? a. Early decelerations b. Late decelerations c. Variable decelerations d. It is always a good idea to change the woman's position.

a. Early decelerations

In assisting with the two factors that have an effect on fetal status (i.e., pushing and positioning), nurses should: a. Encourage the woman's cooperation in avoiding the supine position. b. Advise the woman to avoid the semi-Fowler position. c. Encourage the woman to hold her breath and tighten her abdominal muscles to produce a vaginal response. d. Instruct the woman to open her mouth and close her glottis, letting air escape after the push.

a. Encourage the woman's cooperation in avoiding the supine position.

When managing the care of a woman in the second stage of labor, the nurse uses various measures to enhance the progress of fetal descent. These measures include: a. Encouraging the woman to try various upright positions, including squatting and standing. b. Telling the woman to start pushing as soon as her cervix is fully dilated. c. Continuing an epidural anesthetic so pain is reduced and the woman can relax. d. Coaching the woman to use sustained, 10- to 15-second, closed-glottis bearing-down efforts with each contraction.

a. Encouraging the woman to try various upright positions, including squatting and standing.

An infant with severe meconium aspiration syndrome (MAS) is not responding to conventional treatment. Which highly technical method of treatment may be necessary for an infant who does not respond to conventional treatment? a. Extracorporeal membrane oxygenation b. Respiratory support with a ventilator c. Insertion of a laryngoscope and suctioning of the trachea d. Insertion of an endotracheal tube

a. Extracorporeal membrane oxygenation

Which description of the four stages of labor is correct for both definition and duration? a. First stage: onset of regular uterine contractions to full dilation; less than 1 hour to 20 hours b. Second stage: full effacement to 4 to 5 cm; visible presenting part; 1 to 2 hours c. Third state: active pushing to birth; 20 minutes (multiparous women), 50 minutes (first-timer) d. Fourth stage: delivery of the placenta to recovery; 30 minutes to 1 hour

a. First stage: onset of regular uterine contractions to full dilation; less than 1 hour to 20 hours

The nurse is using the Ballard scale to determine the gestational age of a newborn. Which assessment finding is consistent with a gestational age of 40 weeks? a. Flexed posture b. Abundant lanugo c. Smooth, pink skin with visible veins d. Faint red marks on the soles of the feet

a. Flexed posture

Nurses providing nutritional instruction should be cognizant of the uniqueness of human milk. Which statement is correct? a. Frequent feedings during predictable growth spurts stimulate increased milk production. b. The milk of preterm mothers is the same as the milk of mothers who gave birth at term. c. The milk at the beginning of the feeding is the same as the milk at the end of the feeding. d. Colostrum is an early, less concentrated, less rich version of mature milk.

a. Frequent feedings during predictable growth spurts stimulate increased milk production.

Leopold maneuvers would be an inappropriate method of assessment to determine: a. Gender of the fetus. b. Number of fetuses. c. Fetal lie and attitude. d. Degree of the presenting parts descent into the pelvis.

a. Gender of the fetus.

The most important nursing action in preventing neonatal infection is: a. Good handwashing. b. Isolation of infected infants. c. Separate gown technique. d. Standard Precautions.

a. Good handwashing.

Which breastfeeding patient is most likely to have severe afterbirth pains and request a narcotic analgesic? a. Gravida 5, para 5 b. Woman who is bottle-feeding her first child c. Primipara who delivered a 7-lb boy d. Woman who wishes to breastfeed as soon as her baby is out of the neonatal intensive care unit

a. Gravida 5, para 5

In the recovery room, if a woman is asked either to raise her legs (knees extended) off the bed or to flex her knees, place her feet flat on the bed, and raise her buttocks well off the bed, most likely she is being tested to see whether she: a. Has recovered from epidural or spinal anesthesia. b. Has hidden bleeding underneath her. c. Has regained some flexibility. d. Is a candidate to go home after 6 hours.

a. Has recovered from epidural or spinal anesthesia.

A patient whose cervix is dilated to 5 cm is considered to be in which phase of labor? a. Latent phase b. Active phase c. Second stage d. Third stage

b. Active phase

In assessing a woman for pain and discomfort management during labor, a nurse most likely would: a. Have the woman use a visual analog scale (VAS) to determine her level of pain. b. Note drowsiness as a sign that the medications were working. c. Interpret a womans fist clenching as an indication that she is angry at her male partner and the physician. d. Evaluate the womans skin turgor to see whether she needs a gentle oil massage.

a. Have the woman use a visual analog scale (VAS) to determine her level of pain.

What is an essential part of nursing care for the laboring woman? a. Helping the woman manage the pain b. Eliminating the pain associated with labor c. Sharing personal experiences regarding labor and delivery to decrease her anxiety d. Feeling comfortable with the predictable nature of intrapartum care

a. Helping the woman manage the pain

In the assessment of a preterm infant, the nurse notices continued respiratory distress even though oxygen and ventilation have been provided. The nurse should suspect: a. Hypovolemia and/or shock. b. A nonneutral thermal environment. c. Central nervous system injury. d. Pending renal failure.

a. Hypovolemia and/or shock.

The nurse is discussing storage of breast milk with a mother whose infant is preterm and in the special care unit. What statement would indicate that the mother needs additional teaching? a. I can store my breast milk in the refrigerator for 3 months. b. I can store my breast milk in the freezer for 3 months. c. I can store my breast milk at room temperature for 8 hours. d. I can store my breast milk in the refrigerator for 3 to 5 days.

a. I can store my breast milk in the refrigerator for 3 months.

Nursing follow-up care often includes home visits for the new mother and her infant. Which information related to home visits is correct? a. Ideally, the visit is scheduled within 72 hours after discharge. b. Home visits are available in all areas. c. Visits are completed within a 30-minute time frame. d. Blood draws are not a part of the home visit.

a. Ideally, the visit is scheduled within 72 hours after discharge.

With regard to injuries to the infants plexus during labor and birth, nurses should be aware that: a. If the nerves are stretched with no avulsion, they should recover completely in 3 to 6 months. b. Erb palsy is damage to the lower plexus. c. Parents of children with brachial palsy are taught to pick up the child from under the axillae. d. Breastfeeding is not recommended for infants with facial nerve paralysis until the condition resolves.

a. If the nerves are stretched with no avulsion, they should recover completely in 3 to 6 months.

A hospital has a number of different perineal pads available for use. A nurse is observed soaking several of them and writing down what she sees. This activity indicates that the nurse is trying to: a. Improve the accuracy of blood loss estimation, which usually is a subjective assessment. b. Determine which pad is best. c. Demonstrate that other nurses usually underestimate blood loss. d. Reveal to the nurse supervisor that one of them needs some time off.

a. Improve the accuracy of blood loss estimation, which usually is a subjective assessment.

The nurse would expect which maternal cardiovascular finding during labor? a. Increased cardiac output b. Decreased pulse rate c. Decreased white blood cell (WBC) count d. Decreased blood pressure

a. Increased cardiac output

A pregnant woman wants to breastfeed her infant; however, her husband is not convinced that there are any scientific reasons to do so. The nurse can give the couple printed information comparing breastfeeding and bottle-feeding. Which statement is most accurate? Bottle-feeding using commercially prepared infant formulas: a. Increases the risk that the infant will develop allergies. b. Helps the infant sleep through the night. c. Ensures that the infant is getting iron in a form that is easily absorbed. d. Requires that multivitamin supplements be given to the infant.

a. Increases the risk that the infant will develop allergies.

Which infant would be more likely to have Rh incompatibility? a. Infant of an Rh-negative mother and a father who is Rh positive and homozygous for the Rh factor b. Infant who is Rh negative and whose mother is Rh negative c. Infant of an Rh-negative mother and a father who is Rh positive and heterozygous for the Rh factor d. Infant who is Rh positive and whose mother is Rh positive

a. Infant of an Rh-negative mother and a father who is Rh positive and homozygous for the Rh factor

Which concerns about parenthood are often expressed by visually impaired mothers (Select all that apply)? a. Infant safety b. Transportation c. The ability to care for the infant d. Missing out visually e. Needing extra time for parenting activities to accommodate the visual limitations

a. Infant safety b. Transportation d. Missing out visually e. Needing extra time for parenting activities to accommodate the visual limitations

According to the recommendations of the American Academy of Pediatrics on infant nutrition: a. Infants should be given only human milk for the first 6 months of life. b. Infants fed on formula should be started on solid food sooner than breastfed infants. c. If infants are weaned from breast milk before 12 months, they should receive cows milk, not formula. d. After 6 months mothers should shift from breast milk to cows milk.

a. Infants should be given only human milk for the first 6 months of life.

When planning care for a laboring woman whose membranes have ruptured, the nurse recognizes that the woman's risk for _________________________ has increased. a. Intrauterine infection b. Hemorrhage c. Precipitous labor d. Supine hypotension

a. Intrauterine infection

What PPH conditions are considered medical emergencies that require immediate treatment? a. Inversion of the uterus and hypovolemic shock b. Hypotonic uterus and coagulopathies c. Subinvolution of the uterus and idiopathic thrombocytopenic purpura d. Uterine atony and disseminated intravascular coagulation

a. Inversion of the uterus and hypovolemic shock

In many hospitals new mothers are routinely presented with gift bags containing samples of infant formula. This practice: a. Is inconsistent with the Baby Friendly Hospital Initiative. b. Promotes longer periods of breastfeeding. c. Is perceived as supportive to both bottle-feeding and breastfeeding mothers. d. Is associated with earlier cessation of breastfeeding.

a. Is inconsistent with the Baby Friendly Hospital Initiative.

With regard to the process of augmentation of labor, the nurse should be aware that it: a. Is part of the active management of labor that is instituted when the labor process is unsatisfactory. b. Relies on more invasive methods when oxytocin and amniotomy have failed. c. Is a modern management term to cover up the negative connotations of forceps-assisted birth. d. Uses vacuum cups.

a. Is part of the active management of labor that is instituted when the labor process is unsatisfactory.

The most critical nursing action in caring for the newborn immediately after birth is: a. Keeping the newborns airway clear. b. Fostering parent-newborn attachment. c. Drying the newborn and wrapping the infant in a blanket. d. Administering eye drops and vitamin K.

a. Keeping the newborns airway clear.

During which phase of maternal adjustment will the mother relinquish the baby of her fantasies and accept the real baby? a. Letting go b. Taking hold c. Taking in d. Taking on

a. Letting go

Signs that precede labor include (Select all that apply): a. Lightening. b. Exhaustion. c. Bloody show. d. Rupture of membranes. e. Decreased fetal movement.

a. Lightening c. Exhaustion. d. Rupture of membranes.

An infant at 26 weeks of gestation arrives intubated from the delivery room. The nurse weighs the infant, places him under the radiant warmer, and attaches him to the ventilator at the prescribed settings. A pulse oximeter and cardiorespiratory monitor are placed. The pulse oximeter is recording oxygen saturations of 80%. The prescribed saturations are 92%. The nurses most appropriate action would be to: a. Listen to breath sounds and ensure the patency of the endotracheal tube, increase oxygen, and notify a physician. b. Continue to observe and make no changes until the saturations are 75%. c. Continue with the admission process to ensure that a thorough assessment is completed. d. Notify the parents that their infant is not doing well.

a. Listen to breath sounds and ensure the patency of the endotracheal tube, increase oxygen, and notify a physician.

Nursing care measures are commonly offered to women in labor. Which nursing measure reflects application of the gate-control theory? a. Massaging the woman's back b. Changing the woman's position c. Giving the prescribed medication d. Encouraging the woman to rest between contractions

a. Massaging the woman's back

A collection of blood between the skull bone and its periosteum is known as a cephalhematoma. To reassure the new parents whose infant develops such a soft bulge, it is important that the nurse be aware that this condition: a. May occur with spontaneous vaginal birth. b. Happens only as the result of a forceps or vacuum delivery. c. Is present immediately after birth. d. Will gradually absorb over the first few months of life.

a. May occur with spontaneous vaginal birth.

A pregnant woman was admitted for induction of labor at 43 weeks of gestation with sure dates. A nonstress test (NST) in the obstetricians office revealed a nonreactive tracing. On artificial rupture of membranes, thick, meconium-stained fluid was noted. The nurse caring for the infant after birth should anticipate: a. Meconium aspiration, hypoglycemia, and dry, cracked skin. b. Excessive vernix caseosa covering the skin, lethargy, and respiratory distress syndrome. c. Golden yellow- to green stainedskin and nails, absence of scalp hair, and an increased amount of subcutaneous fat. d. Hyperglycemia, hyperthermia, and an alert, wide-eyed appearance.

a. Meconium aspiration, hypoglycemia, and dry, cracked skin.

A woman in active labor receives an analgesic opioid agonist. Which medication relieves severe, persistent, or recurrent pain; creates a sense of well-being; overcomes inhibitory factors; and may even relax the cervix but should be used cautiously in women with cardiac disease? a. Meperidine (Demerol) b. Promethazine (Phenergan) c. Butorphanol tartrate (Stadol) d. Nalbuphine (Nubain)

a. Meperidine (Demerol)

As the United States and Canada continue to become more culturally diverse, it is increasingly important for the nursing staff to recognize a wide range of varying cultural beliefs and practices. Nurses need to develop respect for these culturally diverse practices and learn to incorporate these into a mutually agreed on plan of care. Although it is common practice in the United States for the father of the baby to be present at the birth, in many societies this is not the case. When implementing care, the nurse would anticipate that a woman from which country would have the father of the baby in attendance? a. Mexico b. China c. Iran d. India

a. Mexico

A careful review of the literature on the various recreational and illicit drugs reveals that: a. More longer-term studies are needed to assess the lasting effects on infants when mothers have taken or are taking illegal drugs. b. Heroin and methadone cross the placenta; marijuana, cocaine, and phencyclidine (PCP) do not. c. Mothers should discontinue heroin use (detox) any time they can during pregnancy. d. Methadone withdrawal for infants is less severe and shorter than heroin withdrawal.

a. More longer-term studies are needed to assess the lasting effects on infants when mothers have taken or are taking illegal drugs.

With regard to nerve block analgesia and anesthesia, nurses should be aware that: a. Most local agents are related chemically to cocaine and end in the suffix -caine. b. Local perineal infiltration anesthesia is effective when epinephrine is added, but it can be injected only once. c. A pudendal nerve block is designed to relieve the pain from uterine contractions. d. A pudendal nerve block, if done correctly, does not significantly lessen the bearing-down reflex.

a. Most local agents are related chemically to cocaine and end in the suffix -caine.

Rho immune globulin will be ordered postpartum if which situation occurs? a. Mother Rh-, baby Rh+ b. Mother Rh-, baby Rh- c. Mother Rh+, baby Rh+ d. Mother Rh+, baby Rh-

a. Mother Rh-, baby Rh+

When the infants behaviors and characteristics call forth a corresponding set of maternal behaviors and characteristics, this is called: a. Mutuality. b. Bonding. c. Claiming. d. Acquaintance.

a. Mutuality.

Several changes in the integumentary system that appear during pregnancy disappear after birth, although not always completely. What change is almost certain to be completely reversed? a. Nail brittleness b. Darker pigmentation of the areolae and linea nigra c. Striae gravidarum on the breasts, abdomen, and thighs d. Spider nevi

a. Nail brittleness

Hearing loss is one of the genetic disorders included in the universal screening program. Auditory screening of all newborns within the first month of life is recommended by the American Academy of Pediatrics. Reasons for having this testing performed include (Select all that apply): a. Prevention or reduction of developmental delay. b. Reassurance for concerned new parents. c. Early identification and treatment. d. Helping the child communicate better. e. Recommendation by the Joint Committee on Infant Hearing.

a. Prevention or reduction of developmental delay. c. Early identification and treatment. d. Helping the child communicate better. e. Recommendation by the Joint Committee on Infant Hearing.

In relation to primary and secondary powers, the maternity nurse comprehends that: a. Primary powers are responsible for effacement and dilation of the cervix. b. Effacement generally is well ahead of dilation in women giving birth for the first time; they are closer together in subsequent pregnancies. c. Scarring of the cervix caused by a previous infection or surgery may make the delivery a bit more painful, but it should not slow or inhibit dilation. d. Pushing in the second stage of labor is more effective if the woman can breathe deeply and control some of her involuntary needs to push, as the nurse directs.

a. Primary powers are responsible for effacement and dilation of the cervix.

Infants born between 34 0/7 and 36 6/7 weeks of gestation are called late-preterm infants because they have many needs similar to those of preterm infants. Because they are more stable than early-preterm infants, they may receive care that is much like that of a full-term baby. The mother-baby or nursery nurse knows that these babies are at increased risk for (Select all that apply): a. Problems with thermoregulation b. Cardiac distress c. Hyperbilirubinemia d. Sepsis e. Hyperglycemia

a. Problems with thermoregulation c. Hyperbilirubinemia d. Sepsis

The nurse providing newborn stabilization must be aware that the primary side effect of maternal narcotic analgesia in the newborn is: a. Respiratory depression. b. Bradycardia. c. Acrocyanosis. d. Tachypne.

a. Respiratory depression.

Women who have participated in childbirth education classes often bring a birth bag or Lamaze bag with them to the hospital. These items often assist in reducing stress and providing comfort measures. The nurse caring for women in labor should be aware of common items that a client may bring, including (Select all that apply): a. Rolling pin. b. Tennis balls. c. Pillow. d. Stuffed animal or photo. e. Candles.

a. Rolling pin. b. Tennis balls. c. Pillow. d. Stuffed animal or photo.

The laboratory results for a postpartum woman are as follows: blood type, A; Rh status, positive; rubella titer, 1:8 (EIA 0.8); hematocrit, 30%. How would the nurse best interpret these data? a. Rubella vaccine should be given. b. A blood transfusion is necessary. c. Rh immune globulin is necessary within 72 hours of birth. d. A Kleihauer-Betke test should be performed.

a. Rubella vaccine should be given.

Induction of labor is considered an acceptable obstetric procedure if it is in the best interest to deliver the fetus. The charge nurse on the labor and delivery unit is often asked to schedule patients for this procedure and therefore must be cognizant of the specific conditions appropriate for labor induction. These include (Select all that apply): a. Rupture of membranes at or near term. b. Convenience of the woman or her physician. c. Chorioamnionitis (inflammation of the amniotic sac). d. Post-term pregnancy. e. Fetal death.

a. Rupture of membranes at or near term. c. Chorioamnionitis. d. Post-term pregnancy. e. Fetal death.

Nurses should be aware of the differences experience can make in labor pain such as: a. Sensory pain for nulliparous women often is greater than for multiparous women during early labor. b. Affective pain for nulliparous women usually is less than for multiparous women throughout the first stage of labor. c. Women with a history of substance abuse experience more pain during labor. d. Multiparous women have more fatigue from labor and therefore experience more pain.

a. Sensory pain for nulliparous women often is greater than for multiparous women during early labor.

Which factors influence cervical dilation (Select all that apply) ? a. Strong uterine contractions b. The force of the presenting fetal part against the cervix c. The size of the female pelvis d. The pressure applied by the amniotic sac e. Scarring of the cervix

a. Strong uterine contractions b. the force of the presenting fetal part against the cervix d. The pressure applied by the amniotic sac e. Scarring of the cervix

The perinatal nurse caring for the postpartum woman understands that late postpartum hemorrhage (PPH) is most likely caused by: a. Subinvolution of the placental site. b. Defective vascularity of the decidua. c. Cervical lacerations. d. Coagulation disorders.

a. Subinvolution of the placental site.

A premature infant with respiratory distress syndrome receives artificial surfactant. How would the nurse explain surfactant therapy to the parents? a. Surfactant improves the ability of your baby's lungs to exchange oxygen and carbon dioxide. b. The drug keeps your baby from requiring too much sedation. c. Surfactant is used to reduce episodes of periodic apnea. d. Your baby needs this medication to fight a possible respiratory tract infection.

a. Surfactant improves the ability of your baby's lungs to exchange oxygen and carbon dioxide.

Pain should be assessed regularly in all newborn infants. If the infant is displaying physiologic or behavioral cues indicating pain, measures should be taken to manage the pain. Examples of nonpharmacologic pain management techniques include (Select all that apply): a. Swaddling. b. Nonnutritive sucking. c. Skin-to-skin contact with the mother. d. Sucrose. e. Acetaminophen.

a. Swaddling. b. Nonnutritive sucking. c. Skin-to-skin contact with the mother. d. Sucrose.

A woman gave birth to a 7-pound, 6-ounce infant girl 1 hour ago. The birth was vaginal, and the estimated blood loss (EBL) was approximately 1500 mL. When assessing the womans vital signs, the nurse would be concerned to see: a. Temperature 37.9 C, heart rate 120, respirations 20, blood pressure (BP) 90/50. b. Temperature 37.4 C, heart rate 88, respirations 36, BP 126/68. c. Temperature 38 C, heart rate 80, respirations 16, BP 110/80. d. Temperature 36.8 C, heart rate 60, respirations 18, BP 140/90.

a. Temperature 37.9 C, heart rate 120, respirations 20, blood pressure (BP) 90/50.

A newborn is placed under a radiant heat warmer, and the nurse evaluates the infants body temperature every hour. Maintaining the newborns body temperature is important for preventing: a. Respiratory depression. b. Cold stress. c. Tachycardia. d. Vasoconstriction.

b. Cold stress.

A first-time father is changing the diaper of his 1-day-old daughter. He asks the nurse, What is this black, sticky stuff in her diaper? The nurses best response is: a. That's meconium, which is your baby's first stool. Its normal. b. That's transitional stool. c. That means your baby is bleeding internally. d. Oh, don't worry about that. Its okay.

a. That's meconium, which is your baby's first stool. Its normal.

A means of controlling the birth of the fetal head with a vertex presentation is: a. The Ritgen maneuver. b. Fundal pressure. c. The lithotomy position. d. The De Lee apparatus.

a. The Ritgen maneuver.

When using intermittent auscultation (IA) to assess uterine activity, the nurse should be cognizant that: a. The examiners hand should be placed over the fundus before, during, and after contractions. b. The frequency and duration of contractions is measured in seconds for consistency. c. Contraction intensity is given a judgment number of 1 to 7 by the nurse and client together. d. The resting tone between contractions is described as either placid or turbulent.

a. The examiners hand should be placed over the fundus before, during, and after contractions.

With regard to the respiratory development of the newborn, nurses should be aware that: a. The first gasping breath is an exaggerated respiratory reaction within 1 minute of birth. b. Newborns must expel the fluid from the respiratory system within a few minutes of birth. c. Newborns are instinctive mouth breathers. d. Seesaw respirations are no cause for concern in the first hour after birth.

a. The first gasping breath is an exaggerated respiratory reaction within 1 minute of birth.

Which finding 12 hours after birth requires further assessment? a. The fundus is palpable two fingerbreadths above the umbilicus. b. The fundus is palpable at the level of the umbilicus. c. The fundus is palpable one fingerbreadth below the umbilicus. Test Bank - Maternal Child Nursing Care 5e (Perry, 2014) 301 d. The fundus is palpable two fingerbreadths below the umbilicus.

a. The fundus is palpable two fingerbreadths above the umbilicus.

The nurse should immediately alert the physician when: a. The infant is dusky and turns cyanotic when crying. b. Acrocyanosis is present at age 1 hour. c. The infants blood glucose level is 45 mg/dL. d. The infant goes into a deep sleep at age 1 hour.

a. The infant is dusky and turns cyanotic when crying.

The best reason for recommending formula over breastfeeding is that: a. The mother has a medical condition or is taking drugs that could be passed along to the infant via breast milk. b. The mother lacks confidence in her ability to breastfeed. c. Other family members or care providers also need to feed the baby. d. The mother sees bottle-feeding as more convenient.

a. The mother has a medical condition or is taking drugs that could be passed along to the infant via breast milk.

As related to the normal functioning of the renal system in newborns, nurses should be aware that: a. The pediatrician should be notified if the newborn has not voided in 24 hours. b. Breastfed infants likely will void more often during the first days after birth. c. Brick dust or blood on a diaper is always cause to notify the physician. d. Weight loss from fluid loss and other normal factors should be made up in 4 to 7 days.

a. The pediatrician should be notified if the newborn has not voided in 24 hours.

When assessing a multiparous woman who has just given birth to an 8-pound boy, the nurse notes that the woman's fundus is firm and has become globular in shape. A gush of dark red blood comes from her vagina. The nurse concludes that: a. The placenta has separated. b. A cervical tear occurred during the birth. c. The woman is beginning to hemorrhage. d. Clots have formed in the upper uterine segment.

a. The placenta has separated.

The mother-baby nurse is able to recognize reciprocal attachment behavior. This refers to: a. The positive feedback an infant exhibits toward parents during the attachment process. b. Behavior during the sensitive period when the infant is in the quiet alert stage. c. Unidirectional behavior exhibited by the infant, initiated and enhanced by eye contact. d. Behavior by the infant during the sensitive period to elicit feelings of falling in love from the parents.

a. The positive feedback an infant exhibits toward parents during the attachment process.

To help clients manage discomfort and pain during labor, nurses should be aware that: a. The predominant pain of the first stage of labor is the visceral pain located in the lower portion of the abdomen. b. Referred pain is the extreme discomfort between contractions. c. The somatic pain of the second stage of labor is more generalized and related to fatigue. d. Pain during the third stage is a somewhat milder version of the second stage.

a. The predominant pain of the first stage of labor is the visceral pain located in the lower portion of the abdomen.

Nurses can assist parents who are trying to decide whether their son should be circumcised by explaining: a. The pros and cons of the procedure during the prenatal period. b. That the American Academy of Pediatrics (AAP) recommends that all newborn boys be routinely circumcised. c. That circumcision is rarely painful and any discomfort can be managed without medication. d. That the infant will likely be alert and hungry shortly after the procedure.

a. The pros and cons of the procedure during the prenatal period.

Fetal well-being during labor is assessed by: a. The response of the fetal heart rate (FHR) to uterine contractions (UCs). b. Maternal pain control. c. Accelerations in the FHR. d. An FHR above 110 beats/min.

a. The response of the fetal heart rate (FHR) to uterine contractions (UCs).

With regard to umbilical cord care, nurses should be aware that: a. The stump can easily become infected. b. A nurse noting bleeding from the vessels of the cord should immediately call for assistance. c. The cord clamp is removed at cord separation. d. The average cord separation time is 5 to 7 days.

a. The stump can easily become infected.

With regard to a womans intake and output during labor, nurses should be aware that: a. The tradition of restricting the laboring woman to clear liquids and ice chips is being challenged because regional anesthesia is used more often than general anesthesia. b. Intravenous (IV) fluids usually are necessary to ensure that the laboring woman stays hydrated. c. Routine use of an enema empties the rectum and is very helpful for producing a clean, clear delivery. d. When a nulliparous woman experiences the urge to defecate, it often means birth will follow quickly.

a. The tradition of restricting the laboring woman to clear liquids and ice chips is being challenged because regional anesthesia is used more often than general anesthesia.

A primiparous woman is to be discharged from the hospital tomorrow with her infant girl. Which behavior indicates a need for further intervention by the nurse before the woman can be discharged? a. The woman leaves the infant on her bed while she takes a shower. b. The woman continues to hold and cuddle her infant after she has fed her. c. The woman reads a magazine while her infant sleeps. d. The woman changes her infants diaper and then shows the nurse the contents of the diaper.

a. The woman leaves the infant on her bed while she takes a shower.

Before the physician performs an external version, the nurse should expect an order for a: a. Tocolytic drug. b. Contraction stress test (CST). c. Local anesthetic. d. Foley catheter.

a. Tocolytic drug.

A parent who has a hearing impairment is presented with a number of challenges in parenting. Which nursing approaches are appropriate for working with hearing-impaired new parents (Select all that apply)? a. Use devices that transform sound into light. b. Assume that the patient knows sign language. c. Speak quickly and loudly. d. Ascertain whether the patient can read lips before teaching. e. Written messages aid in communication.

a. Use devices that transform sound into light. d. Ascertain whether the patient can read lips before teaching. e. Written messages aid in communication.

The perinatal nurse is caring for a woman in the immediate postbirth period. Assessment reveals that the woman is experiencing profuse bleeding. The most likely etiology for the bleeding is: a. Uterine atony. b. Uterine inversion. c. Vaginal hematoma. d. Vaginal laceration.

a. Uterine atony.

The nurse providing care for the laboring woman should understand that amnioinfusion is used to treat: a. Variable decelerations. b. Late decelerations. c. Fetal bradycardia. d. Fetal tachycardia.

a. Variable decelerations.

The cheeselike, whitish substance that fuses with the epidermis and serves as a protective coating is called: a. Vernix caseosa. b. Surfactant. c. Caput succedaneum. d. Acrocyanosis.

a. Vernix caseosa.

The exact cause of preterm labor is unknown and believed to be multifactorial. Infection is thought to be a major factor in many preterm labors. Select the type of infection that has not been linked to preterm births. a. Viral b. Periodontal c. Cervical d. Urinary tract

a. Viral

The nurse caring for the newborn should be aware that the sensory system least mature at the time of birth is: a. Vision. b. Hearing. c. Smell. d. Taste.

a. Vision.

In the United States the en face position is preferred immediately after birth. Nurses can facilitate this process by all of these actions except: a. Washing both the infants face and the mothers face. b. Placing the infant on the mothers abdomen or breast with their heads on the same plane. c. Dimming the lights. d. Delaying the instillation of prophylactic antibiotic ointment in the infants eyes.

a. Washing both the infants face and the mothers face.

The nurse notes that a Vietnamese woman does not cuddle or interact with her newborn other than to feed him, change his diapers or soiled clothes, and put him to bed. In evaluating the woman's behavior with her infant, the nurse realizes that: a. What appears to be a lack of interest in the newborn is in fact the Vietnamese way of demonstrating intense love by attempting to ward off evil spirits. b. The woman is inexperienced in caring for newborns. c. The woman needs a referral to a social worker for further evaluation of her parenting behaviors once she goes home with the newborn. d. Extra time needs to be planned for assisting the woman in bonding with her newborn.

a. What appears to be a lack of interest in the newborn is in fact the Vietnamese way of demonstrating intense love by attempting to ward off evil spirits.

The class of drugs known as opioid analgesics (butorphanol, nalbuphine) is not suitable for administration to women with known opioid dependence. The antagonistic activity could precipitate withdrawal symptoms (abstinence syndrome) in both mothers and newborns. Signs of opioid/narcotic withdrawal in the mother would include (Select all that apply): a. Yawning, runny nose. b. Increase in appetite. c. Chills and hot flashes. d. Constipation. e. Irritability, restlessness.

a. Yawning, runny nose. c. Chills and hot flashes. e. Irritability, restlessness.

With regard to spinal and epidural (block) anesthesia, nurses should know that: a. This type of anesthesia is commonly used for cesarean births but is not suitable for vaginal births. b. A high incidence of after-birth headache is seen with spinal blocks. c. Epidural blocks allow the woman to move freely. d. Spinal and epidural blocks are never used together.

b. A high incidence of after-birth headache is seen with spinal blocks.

The nerve block used in labor that provides anesthesia to the lower vagina and perineum is called: a. An epidural. b. A pudendal. c. A local. d. A spinal block.

b. A pudendal

Which woman is most likely to experience strong afterpains? a. A woman who experienced oligohydramnios b. A woman who is a gravida 4, para 4-0-0-4 c. A woman who is bottle-feeding her infant d. A woman whose infant weighed 5 pounds, 3 ounces

b. A woman who is a gravida 4, para 4-0-0-4

Which woman is at greatest risk for early postpartum hemorrhage (PPH)? a. A primiparous woman (G 2 P 1 0 0 1) being prepared for an emergency cesarean birth for fetal distress b. A woman with severe preeclampsia who is receiving magnesium sulfate and whose labor is being induced c. A multiparous woman (G 3 P 2 0 0 2) with an 8-hour labor d. A primigravida in spontaneous labor with preterm twins

b. A woman with severe preeclampsia who is receiving magnesium sulfate and whose labor is being induced

Necrotizing enterocolitis (NEC) is an inflammatory disease of the gastrointestinal mucosa. The signs of NEC are nonspecific. Some generalized signs include: a. Hypertonia, tachycardia, and metabolic alkalosis. b. Abdominal distention, temperature instability, and grossly bloody stools. c. Hypertension, absence of apnea, and ruddy skin color. d. Scaphoid abdomen, no residual with feedings, and increased urinary output.

b. Abdominal distention, temperature instability, and grossly bloody stools.

The role of the nurse with regard to informed consent is to: a. Inform the client about the procedure and have her sign the consent form. b. Act as a client advocate and help clarify the procedure and the options. c. Call the physician to see the client. d. Witness the signing of the consent form.

b. Act as a client advocate and help clarify the procedure and the options.

Nurses can help their clients by keeping them informed about the distinctive stages of labor. What description of the phases of the first stage of labor is accurate? a. Latent: Mild, regular contractions; no dilation; bloody show; duration of 2 to 4 hours b. Active: Moderate, regular contractions; 4- to 7-cm dilation; duration of 3 to 6 hours c. Lull: No contractions; dilation stable; duration of 20 to 60 minutes d. Transition: Very strong but irregular contractions; 8- to 10-cm dilation; duration of 1 to 2 hours

b. Active: Moderate, regular contractions; 4- to 7-cm dilation; duration of 3 to 6 hours

An examiner who discovers unequal movement or uneven gluteal skin folds during the Ortolani maneuver would then: a. Tell the parents that one leg may be longer than the other, but they will equal out by the time the infant is walking. b. Alert the physician that the infant has a dislocated hip. c. Inform the parents and physician that molding has not taken place. d. Suggest that, if the condition does not change, surgery to correct vision problems may be needed.

b. Alert the physician that the infant has a dislocated hip.

Concerning the third stage of labor, nurses should be aware that: a. The placenta eventually detaches itself from a flaccid uterus. b. An expectant or active approach to managing this stage of labor reduces the risk of complications. c. It is important that the dark, roughened maternal surface of the placenta appear before the shiny fetal surface. d. The major risk for women during the third stage is a rapid heart rate.

b. An expectant or active approach to managing this stage of labor reduces the risk of complications.

To prevent the abduction of newborns from the hospital, the nurse should: a. Instruct the mother not to give her infant to anyone except the one nurse assigned to her that day. b. Apply an electronic and identification bracelet to mother and infant. c. Carry the infant when transporting him or her in the halls. d. Restrict the amount of time infants are out of the nursery.

b. Apply an electronic and identification bracelet to mother and infant.

A woman gave birth 48 hours ago to a healthy infant girl. She has decided to bottle-feed. During your assessment you notice that both of her breasts are swollen, warm, and tender on palpation. The woman should be advised that this condition can best be treated by: a. Running warm water on her breasts during a shower. b. Applying ice to the breasts for comfort. c. Expressing small amounts of milk from the breasts to relieve pressure. d. Wearing a loose-fitting bra to prevent nipple irritation.

b. Applying ice to the breasts for comfort.

Plantar creases should be evaluated within a few hours of birth because: a. The newborn has to be footprinted. b. As the skin dries, the creases will become more prominent. c. Heel sticks may be required. d. Creases will be less prominent after 24 hours.

b. As the skin dries, the creases will become more prominent.

A woman delivered a 9-lb, 10-oz baby 1 hour ago. When you arrive to perform her 15-minute assessment, she tells you that she feels all wet underneath. You discover that both pads are completely saturated and that she is lying in a 6-inch-diameter puddle of blood. What is your first action? a. Call for help. b. Assess the fundus for firmness. c. Take her blood pressure. d. Check the perineum for lacerations.

b. Assess the fundus for firmness.

The nurse providing care for a woman with preterm labor who is receiving terbutaline would include which intervention to identify side effects of the drug? a. Assessing deep tendon reflexes (DTRs) b. Assessing for chest discomfort and palpitations c. Assessing for bradycardia d. Assessing for hypoglycemia

b. Assessing for chest discomfort and palpitations

To promote bonding and attachment immediately after delivery, the most important nursing intervention is to: a. Allow the mother quiet time with her infant. b. Assist the mother in assuming an en face position with her newborn. c. Teach the mother about the concepts of bonding and attachment. d. Assist the mother in feeding her baby.

b. Assist the mother in assuming an en face position with her newborn.

The prevalence of urinary incontinence (UI) increases as women age, with more than one third of women in the United States suffering from some form of this disorder. The symptoms of mild to moderate UI can be successfully decreased by a number of strategies. Which of these should the nurse instruct the client to use first? a. Pelvic floor support devices b. Bladder training and pelvic muscle exercises c. Surgery d. Medications

b. Bladder training and pelvic muscle exercises

The breastfeeding mother should be taught a safe method to remove the breast from the baby's mouth. Which suggestion by the nurse is most appropriate? a. Slowly remove the breast from the baby's mouth when the infant has fallen asleep and the jaws are relaxed. b. Break the suction by inserting your finger into the corner of the infants mouth. c. A popping sound occurs when the breast is correctly removed from the infants mouth. d. Elicit the Moro reflex to wake the baby and remove the breast when the baby cries.

b. Break the suction by inserting your finger into the corner of the infants mouth.

Necrotizing enterocolitis (NEC) is an acute inflammatory disease of the gastrointestinal mucosa that can progress to perforation of the bowel. Approximately 2% to 5% of premature infants succumb to this fatal disease. Care is supportive; however, known interventions may decrease the risk of NEC. To develop an optimal plan of care for this infant, the nurse must understand which intervention has the greatest effect on lowering the risk of NEC: a. Early enteral feedings b. Breastfeeding c. Exchange transfusion d. Prophylactic probiotics

b. Breastfeeding

The nurse is explaining the benefits associated with breastfeeding to a new mother. Which statement by the nurse would be inaccurate and provide conflicting information to the patient? a. Women who breastfeed have a decreased risk of breast cancer. b. Breastfeeding is an effective method of birth control. c. Breastfeeding increases bone density. d. Breastfeeding may enhance postpartum weight loss.

b. Breastfeeding is an effective method of birth control.

A tiered system of categorizing FHR has been recommended by regulatory agencies. Nurses, midwives, and physicians who care for women in labor must have a working knowledge of fetal monitoring standards and understand the significance of each category. These categories include (Select all that apply): a. Reassuring. b. Category I. c. Category II. d. Nonreassuring. e. Category III.

b. Category I. c. Category II. e. Category III.

A mother in late middle age who is certain she is not pregnant tells the nurse during an office visit that she has urinary problems and sensations of bearing down and of something in her vagina. The nurse would realize that the client most likely is suffering from: a. Pelvic relaxation. b. Cystoceles and/or rectoceles. c. Uterine displacement. d. Genital fistulas.

b. Cystoceles and/or rectoceles.

When caring for a newly delivered woman, the nurse is aware that the best measure to prevent abdominal distention after a cesarean birth is: a. Rectal suppositories. b. Early and frequent ambulation. c. Tightening and relaxing abdominal muscles. d. Carbonated beverages.

b. Early and frequent ambulation.

With regard to systemic analgesics administered during labor, nurses should be aware that: a. Systemic analgesics cross the maternal blood-brain barrier as easily as they do the fetal blood-brain barrier. b. Effects on the fetus and newborn can include decreased alertness and delayed sucking. c. Intramuscular administration (IM) is preferred over intravenous (IV) administration. d. IV patient-controlled analgesia (PCA) results in increased use of an analgesic.

b. Effects on the fetus and newborn can include decreased alertness and delayed sucking.

In planning for an expected cesarean birth for a woman who has given birth by cesarean previously and who has a fetus in the transverse presentation, which information would the nurse include? a. Because this is a repeat procedure, you are at the lowest risk for complications. b. Even though this is your second cesarean birth, you may wish to review the preoperative and postoperative procedures. c. Because this is your second cesarean birth, you will recover faster. d. You will not need preoperative teaching because this is your second cesarean birth.

b. Even though this is your second cesarean birth, you may wish to review the preoperative and postoperative procedures.

A woman gave birth to a 7-pound, 3-ounce infant boy 2 hours ago. The nurse determines that the woman's bladder is distended because her fundus is now 3 cm above the umbilicus and to the right of the midline. In the immediate postpartum period, the most serious consequence likely to occur from bladder distention is: a. Urinary tract infection. b. Excessive uterine bleeding. c. A ruptured bladder. d. Bladder wall atony.

b. Excessive uterine bleeding.

A woman who is gravida 3 para 2 enters the intrapartum unit. The most important nursing assessments are: a. Contraction pattern, amount of discomfort, and pregnancy history. b. Fetal heart rate, maternal vital signs, and the woman's nearness to birth. c. Identification of ruptured membranes, the woman's gravida and para, and her support person. d. Last food intake, when labor began, and cultural practices the couple desires.

b. Fetal heart rate, maternal vital signs, and the woman's nearness to birth.

A woman gave birth to a healthy 7-pound, 13-ounce infant girl. The nurse suggests that the woman place the infant to her breast within 15 minutes after birth. The nurse knows that breastfeeding is effective during the first 30 minutes after birth because this is the: a. Transition period. b. First period of reactivity. c. Organizational stage. d. Second period of reactivity.

b. First period of reactivity.

Through vaginal examination the nurse determines that a woman is 4 cm dilated, and the external fetal monitor shows uterine contractions every 3.5 to 4 minutes. The nurse would report this as: a. First stage, latent phase. b. First stage, active phase. c. First stage, transition phase. d. Second stage, latent phase.

b. First stage, active phase.

Late in pregnancy, the womans breasts should be assessed by the nurse to identify any potential concerns related to breastfeeding. Some nipple conditions make it necessary to provide intervention before birth. These include(Select all that apply): a. Everted nipples b. Flat nipples c. Inverted nipples d. Nipples that contract when compressed e. Cracked nipples

b. Flat nipples c. Inverted nipples d. Nipples that contract when compressed

A newborn was admitted to the neonatal intensive care unit after being delivered at 29 weeks of gestation to a 28-year-old multiparous, married, Caucasian woman whose pregnancy was uncomplicated until premature rupture of membranes and preterm birth. The newborns parents arrive for their first visit after the birth. The parents walk toward the bedside but remain approximately 5 feet away from the bed. The nurses most appropriate action would be to: a. Wait quietly at the newborns bedside until the parents come closer. b. Go to the parents, introduce himself or herself, and gently encourage the parents to come meet their infant; explain the equipment first, and then focus on the newborn. c. Leave the parents at the bedside while they are visiting so they can have some privacy. d. Tell the parents only about the newborns physical condition, and caution them to avoid touching their baby.

b. Go to the parents, introduce himself or herself, and gently encourage the parents to come meet their infant; explain the equipment first, and then focus on the newborn.

Other early sensual contacts between infant and mother involve sound and smell. Nurses should be aware that, despite what folk wisdom may say: a. High-pitched voices irritate newborns. b. Infants can learn to distinguish their mothers voice from others soon after birth. c. All babies in the hospital smell alike. d. A mothers breast milk has no distinctive odor.

b. Infants can learn to distinguish their mothers voice from others soon after birth.

The parents of a newborn ask the nurse how much the newborn can see. The parents specifically want to know what type of visual stimuli they should provide for their newborn. The nurse responds to the parents by telling them: a. Infants can see very little until about 3 months of age. b. Infants can track their parents eyes and distinguish patterns; they prefer complex patterns. c. The infants eyes must be protected. Infants enjoy looking at brightly colored stripes. d. Its important to shield the newborns eyes. Overhead lights help them see better.

b. Infants can track their parents eyes and distinguish patterns; they prefer complex patterns.

With regard to small for gestational age (SGA) infants and intrauterine growth restrictions (IUGR), nurses should be aware that: a. In the first trimester diseases or abnormalities result in asymmetric IUGR. b. Infants with asymmetric IUGR have the potential for normal growth and development. c. In asymmetric IUGR weight is slightly more than SGA, whereas length and head circumference are somewhat less than SGA. d. Symmetric IUGR occurs in the later stages of pregnancy.

b. Infants with asymmetric IUGR have the potential for normal growth and development.

Infants in whom cephalhematomas develop are at increased risk for: a. Infection. b. Jaundice. c. Caput succedaneum. d. Erythema toxicum.

b. Jaundice.

While caring for the patient who requires an induction of labor, the nurse should be cognizant that: a. Ripening the cervix usually results in a decreased success rate for induction. b. Labor sometimes can be induced with balloon catheters or laminaria tents. c. Oxytocin is less expensive than prostaglandins and more effective but creates greater health risks. d. Amniotomy can be used to make the cervix more favorable for labor.

b. Labor sometimes can be induced with balloon catheters or laminaria tents.

A steady trickle of bright red blood from the vagina in the presence of a firm fundus suggests: a. Uterine atony. b. Lacerations of the genital tract. c. Perineal hematoma. d. Infection of the uterus.

b. Lacerations of the genital tract.

The transition period between intrauterine and extrauterine existence for the newborn: a. Consists of four phases, two reactive and two of decreased responses. b. Lasts from birth to day 28 of life. c. Applies to full-term births only. d. Varies by socioeconomic status and the mothers age.

b. Lasts from birth to day 28 of life.

What correctly matches the type of deceleration with its likely cause? a. Early decelerationumbilical cord compression b. Late decelerationuteroplacental inefficiency c. Variable decelerationhead compression d. Prolonged decelerationcause unknown

b. Late decelerationuteroplacental inefficiency

A new father is ready to take his wife and newborn son home. He proudly tells the nurse who is discharging them that within the next week he plans to start feeding the infant cereal between breastfeeding sessions. The nurse can explain to him that beginning solid foods before 4 to 6 months may: a. Decrease the infants intake of sufficient calories. b. Lead to early cessation of breastfeeding. c. Help the infant sleep through the night. d. Limit the infants growth.

b. Lead to early cessation of breastfeeding.

For women who have a history of sexual abuse, a number of traumatic memories may be triggered during labor. The woman may fight the labor process and react with pain or anger. Alternately, she may become a passive player and emotionally absent herself from the process. The nurse is in a unique position of being able to assist the client to associate the sensations of labor with the process of childbirth and not the past abuse. The nurse can implement a number of care measures to help the client view the childbirth experience in a positive manner. Which intervention would be key for the nurse to use while providing care? a. Telling the client to relax and that it wont hurt much b. Limiting the number of procedures that invade her body c. Reassuring the client that as the nurse you know what is best d. Allowing unlimited care providers to be with the client

b. Limiting the number of procedures that invade her body

A new mother recalls from prenatal class that she should try to feed her newborn daughter when she exhibits feeding readiness cues rather than waiting until her infant is crying frantically. On the basis of this information, this woman should feed her infant about every 2.5 to 3 hours when she: a. Waves her arms in the air. b. Makes sucking motions. c. Has hiccups. d. Stretches her legs out straight.

b. Makes sucking motions.

The postpartum woman who continually repeats the story of her labor, delivery, and recovery experience is: a. Providing others with her knowledge of events. b. Making the birth experience real. c. Taking hold of the events leading to her labor and delivery. d. Accepting her response to labor and delivery.

b. Making the birth experience real.

Which statement is the best rationale for assessing maternal vital signs between contractions? a. During a contraction, assessing fetal heart rates is the priority. b. Maternal circulating blood volume increases temporarily during contractions. c. Maternal blood flow to the heart is reduced during contractions. d. Vital signs taken during contractions are not accurate.

b. Maternal circulating blood volume increases temporarily during contractions.

A maternal indication for the use of vacuum extraction is: a. A wide pelvic outlet. b. Maternal exhaustion. c. A history of rapid deliveries. d. Failure to progress past 0 station.

b. Maternal exhaustion.

The slight overlapping of cranial bones or shaping of the fetal head during labor is called: a. Lightening. b. Molding. c. Ferguson reflex. d. Valsalva maneuver.

b. Molding.

To provide optimal care of infants born to mothers who are substance abusers, nurses should be aware that: a. Infants born to addicted mothers are also addicted. b. Mothers who abuse one substance likely will use or abuse another, thus compounding the infants difficulties. c. The NICU Network Neurobehavioral Scale (NNNS) is designed to assess the damage the mother has done to herself. d. No laboratory procedures are available that can identify the intrauterine drug exposure of the infant.

b. Mothers who abuse one substance likely will use or abuse another, thus compounding the infants difficulties.

Which statement by a newly delivered woman indicates that she knows what to expect about her menstrual activity after childbirth? a. My first menstrual cycle will be lighter than normal and then will get heavier every month thereafter. b. My first menstrual cycle will be heavier than normal and will return to my prepregnant volume within three or four cycles. c. I will not have a menstrual cycle for 6 months after childbirth. d. My first menstrual cycle will be heavier than normal and then will be light for several months after.

b. My first menstrual cycle will be heavier than normal and will return to my prepregnant volume

The process whereby parents awaken the infant to feed every 3 hours during the day and at least every 4 hours at night is: a. Known as demand feeding. b. Necessary during the first 24 to 48 hours after birth. c. Used to set up the supply-meets-demand system. d. A way to control cluster feeding.

b. Necessary during the first 24 to 48 hours after birth.

It is important for the nurse to develop a realistic birth plan with the pregnant woman in her care. The nurse can explain that a major advantage of nonpharmacologic pain management is: a. Greater and more complete pain relief is possible. b. No side effects or risks to the fetus are involved. c. The woman remains fully alert at all times. d. A more rapid labor is likely.

b. No side effects or risks to the fetus are involved.

A woman in labor has just received an epidural block. The most important nursing intervention is to: a. Limit parenteral fluids. b. Monitor the fetus for possible tachycardia. c. Monitor the maternal blood pressure for possible hypotension. d. Monitor the maternal pulse for possible bradycardia.

c. Monitor the maternal blood pressure for possible hypotension.

To care adequately for infants at risk for neonatal bacterial infection, nurses should be aware that: a. Congenital infection progresses more slowly than does nosocomial infection. b. Nosocomial infection can be prevented by effective handwashing; early-onset infections cannot. c. Infections occur with about the same frequency in boy and girl infants, although female mortality is higher. d. The clinical sign of a rapid, high fever makes infection easier to diagnose.

b. Nosocomial infection can be prevented by effective handwashing; early-onset infections cannot.

With regard to the adaptation of other family members, mainly siblings and grandparents, to the newborn, nurses should be aware that: a. Sibling rivalry cannot be dismissed as overblown psychobabble; negative feelings and behaviors can take a long time to blow over. b. Participation in preparation classes helps both siblings and grandparents. c. In the United States paternal and maternal grandparents consider themselves of equal importance and status. d. In the past few decades the number of grandparents providing permanent care to their grandchildren has been declining.

b. Participation in preparation classes helps both siblings and grandparents.

A primary nursing responsibility when caring for a woman experiencing an obstetric hemorrhage associated with uterine atony is to: a. Establish venous access. b. Perform fundal massage. c. Prepare the woman for surgical intervention. d. Catheterize the bladder.

b. Perform fundal massage.

The nurses initial action when caring for an infant with a slightly decreased temperature is to: a. Notify the physician immediately. b. Place a cap on the infants head and have the mother perform kangaroo care. c. Tell the mother that the infant must be kept in the nursery and observed for the next 4 hours. d. Change the formula because this is a sign of formula intolerance.

b. Place a cap on the infants head and have the mother perform kangaroo care.

Maternal hypotension is a potential side effect of regional anesthesia and analgesia. What nursing interventions could you use to raise the clients blood pressure (Select all that apply)? a. Place the woman in a supine position. b. Place the woman in a lateral position. c. Increase intravenous (IV) fluids. d. Administer oxygen. e. Perform a vaginal examination.

b. Place the woman in a lateral position. c. Increase intravenous (IV) fluids. d. Administer oxygen.

To prevent nipple trauma, the nurse should instruct the new mother to: a. Limit the feeding time to less than 5 minutes. b. Position the infant so the nipple is far back in the mouth. c. Assess the nipples before each feeding. d. Wash the nipples daily with mild soap and water.

b. Position the infant so the nipple is far back in the mouth.

Nurses should know some basic definitions concerning preterm birth, preterm labor, and low birth weight. For instance: a. The terms preterm birth and low birth weight can be used interchangeably. b. Preterm labor is defined as cervical changes and uterine contractions occurring between 20 and 37 weeks of pregnancy. c. Low birth weight is anything below 3.7 pounds. d. In the United States early in this century, preterm birth accounted for 18% to 20% of all births.

b. Preterm labor is defined as cervical changes and uterine contractions occurring between 20 and 37 weeks of pregnancy.

The goal of treatment of the infant with phenylketonuria (PKU) is to: a. Cure mental retardation. b. Prevent central nervous system (CNS) damage, which leads to mental retardation. c. Prevent gastrointestinal symptoms. d. Cure the urinary tract infection.

b. Prevent central nervous system (CNS) damage, which leads to mental retardation.

A man calls the nurses station and states that his wife, who delivered 2 days ago, is happy one minute and crying the next. The man says, She was never like this before the baby was born. The nurses initial response could be to: a. Tell him to ignore the mood swings, as they will go away. b. Reassure him that this behavior is normal. c. Advise him to get immediate psychological help for her. d. Instruct him in the signs, symptoms, and duration of postpartum blues.

b. Reassure him that this behavior is normal.

What three measures should the nurse implement to provide intrauterine resuscitation? Select the response that best indicates the priority of actions that should be taken. a. Call the provider, reposition the mother, and perform a vaginal examination. b. Reposition the mother, increase intravenous (IV) fluid, and provide oxygen via face mask. c. Administer oxygen to the mother, increase IV fluid, and notify the care provider. d. Perform a vaginal examination, reposition the mother, and provide oxygen via face mask.

b. Reposition the mother, increase intravenous (IV) fluid, and provide oxygen via face mask.

With regard to the postpartum changes and developments in a womans cardiovascular system, nurses should be aware that: a. Cardiac output, the pulse rate, and stroke volume all return to prepregnancy normal values within a few hours of childbirth. b. Respiratory function returns to nonpregnant levels by 6 to 8 weeks after birth. c. The lowered white blood cell count after pregnancy can lead to false-positive results on tests for infections. d. A hypercoagulable state protects the new mother from thromboembolism, especially after a cesarean birth.

b. Respiratory function returns to nonpregnant levels by 6 to 8 weeks after birth.

In caring for the preterm infant, what complication is thought to be a result of high arterial blood oxygen level? a. Necrotizing enterocolitis (NEC) b. Retinopathy of prematurity (ROP) c. Bronchopulmonary dysplasia (BPD) d. Intraventricular hemorrhage (IVH)

b. Retinopathy of prematurity (ROP)

A first-time mother is concerned about the type of medications she will receive during labor. She is in a fair amount of pain and is nauseous. In addition, she appears to be very anxious. You explain that opioid analgesics often are used with sedatives because: a. The two together work the best for you and your baby. b. Sedatives help the opioid work better, and they also will assist you to relax and relieve your nausea. c. They work better together so you can sleep until you have the baby. d. This is what the doctor has ordered for you.

b. Sedatives help the opioid work better, and they also will assist you to relax and relieve your nausea.

The nurse observes several interactions between a postpartum woman and her new son. What behavior, if exhibited by this woman, would the nurse identify as a possible maladaptive behavior regarding parent-infant attachment? a. Talks and coos to her son b. Seldom makes eye contact with her son c. Cuddles her son close to her d. Tells visitors how well her son is feeding

b. Seldom makes eye contact with her son

With regard to a pregnant woman's anxiety and pain experience, nurses should be aware that: a. Even mild anxiety must be treated. b. Severe anxiety increases tension, which increases pain, which in turn increases fear and anxiety, and so on. c. Anxiety may increase the perception of pain, but it does not affect the mechanism of labor. d. Women who have had a painful labor will have learned from the experience and have less anxiety the second time because of increased familiarity.

b. Severe anxiety increases tension, which increases pain, which in turn increases fear and anxiety, and so on.

The nurse observes that a 15-year-old mother seems to ignore her newborn. A strategy that the nurse can use to facilitate mother-infant attachment in this mother is to: a. Tell the mother she must pay attention to her infant. b. Show the mother how the infant initiates interaction and pays attention to her. c. Demonstrate for the mother different positions for holding her infant while feeding. d. Arrange for the mother to watch a video on parent-infant interaction.

b. Show the mother how the infant initiates interaction and pays attention to her.

The nurse expects to administer an oxytocic (e.g., Pitocin, Methergine) to a woman after expulsion of her placenta to: a. Relieve pain. b. Stimulate uterine contraction. c. Prevent infection. d. Facilitate rest and relaxation.

b. Stimulate uterine contraction.

The nurse has received report regarding her patient in labor. The woman's last vaginal examination was recorded as 3 cm, 30%, and ?2-2. The nurses interpretation of this assessment is that: a. The cervix is effaced 3 cm, it is dilated 30%, and the presenting part is 2 cm above the ischial spines. b. The cervix is 3 cm dilated, it is effaced 30%, and the presenting part is 2 cm above the ischial spines. c. The cervix is effaced 3 cm, it is dilated 30%, and the presenting part is 2 cm below the ischial spines. d. The cervix is dilated 3 cm, it is effaced 30%, and the presenting part is 2 cm below the ischial spines.

b. The cervix is 3 cm dilated, it is effaced 30%, and the presenting part is 2 cm above the ischial spines.

By knowing about variations in infants blood count, nurses can explain to their clients that: a. A somewhat lower than expected red blood cell count could be the result of delay in clamping the umbilical cord. b. The early high white blood cell (WBC) count is normal at birth and should decrease rapidly. c. Platelet counts are higher than in adults for a few months. d. Even a modest vitamin K deficiency means a problem with the ability of the blood to clot properly.

b. The early high white blood cell (WBC) count is normal at birth and should decrease rapidly.

The nurse should be aware that an effective plan to achieve adequate pain relief without maternal risk is most effective if: a. The mother gives birth without any analgesic or anesthetic. b. The mother and family's priorities and preferences are incorporated into the plan. c. The primary health care provider decides the best pain relief for the mother and family. d. The nurse informs the family of all alternative methods of pain relief available in the hospital setting.

b. The mother and family's priorities and preferences are incorporated into the plan.

During the complete physical examination 24 hours after birth: a. The parents are excused to reduce their normal anxiety. b. The nurse can gauge the neonates maturity level by assessing the infants general appearance. c. Once often neglected, blood pressure is now routinely checked. d. When the nurse listens to the heart, the S1 and S2 sounds can be heard; the first sound is somewhat higher in pitch and sharper than the second.

b. The nurse can gauge the neonates maturity level by assessing the infants general appearance.

Nurses alert to signs of the onset of the second stage of labor can be certain that this stage has begun when: a. The woman has a sudden episode of vomiting. b. The nurse is unable to feel the cervix during a vaginal examination. c. Bloody show increases. d. The woman involuntarily bears down.

b. The nurse is unable to feel the cervix during a vaginal examination.

In follow-up appointments or visits with parents and their new baby, it may be useful if the nurse can identify parental behaviors that can either facilitate or inhibit attachment. Which one is a facilitating behavior? a. The parents have difficulty naming the infant. b. The parents hover around the infant, directing attention to and pointing at the infant. c. The parents make no effort to interpret the actions or needs of the infant. d. The parents do not move from fingertip touch to palmar contact and holding.

b. The parents hover around the infant, directing attention to and pointing at the infant.

A new client and her partner arrive on the labor, delivery, recovery, and postpartum unit for the birth of their first child. You apply the electronic fetal monitor (EFM) to the woman. Her partner asks you to explain what is printing on the graph, referring to the EFM strip. He wants to know what the baby's heart rate should be. Your best response is: a. Don't worry about that machine; that's my job. b. The top line graphs the baby's heart rate. Generally the heart rate is between 110 and 160. The heart rate will fluctuate in response to what is happening during labor. c. The top line graphs the baby's heart rate, and the bottom line lets me know how strong the contractions are. d. Your doctor will explain all of that later.

b. The top line graphs the baby's heart rate. Generally the heart rate is between 110 and 160. The heart rate will fluctuate in response to what is happening during labor.

A newborn in the neonatal intensive care unit (NICU) is dying as a result of a massive infection. The parents speak to the neonatologist, who informs them of their sons prognosis. When the father sees his son, he says, He looks just fine to me. I cant understand what all this is about. The most appropriate response by the nurse would be: a. Didn't the doctor tell you about your sons problems? b. This must be a difficult time for you. Tell me how you're doing. c. To stand beside him quietly. d. You'll have to face up to the fact that he is going to die sooner or later.

b. This must be a difficult time for you. Tell me how you're doing.

The primary difference between the labor of a nullipara and that of a multipara is the: a. Amount of cervical dilation. b. Total duration of labor. c. Level of pain experienced. d. Sequence of labor mechanisms.

b. Total duration of labor.

The nurse providing care for the laboring woman realizes that variable fetal heart rate (FHR) decelerations are caused by: a. Altered fetal cerebral blood flow. b. Umbilical cord compression. c. Uteroplacental insufficiency. d. Fetal hypoxemia.

b. Umbilical cord compression.

The nurse recognizes that uterine hyperstimulation with oxytocin requires emergency interventions. What clinical cues would alert the nurse that the woman is experiencing uterine hyperstimulation (Select all that apply)? a. Uterine contractions lasting <90 seconds and occurring >2 minutes in frequency b. Uterine contractions lasting >90 seconds and occurring <2 minutes in frequency c. Uterine tone <20 mm Hg d. Uterine tone >20 mm Hg e. Increased uterine activity accompanied by a nonreassuring fetal heart rate (FHR) and pattern

b. Uterine contractions lasting >90 seconds and occurring <2 minutes in frequency d. Uterine tone >20 mmHg e. Increased uterine activity accompanied by a nonreassuring fetal heart rate (FHR) and pattern.

Why is continuous electronic fetal monitoring usually used when oxytocin is administered? a. The mother may become hypotensive. b. Uteroplacental exchange may be compromised. c. Maternal fluid volume deficit may occur. d. Fetal chemoreceptors are stimulated.

b. Uteroplacental exchange may be compromised.

Which description of postpartum restoration or healing times is accurate? a. The cervix shortens, becomes firm, and returns to form within a month postpartum. b. Vaginal rugae reappear by 3 weeks postpartum. c. Most episiotomies heal within a week. d. Hemorrhoids usually decrease in size within 2 weeks of childbirth.

b. Vaginal rugae reappear by 3 weeks postpartum.

When working with parents who have some form of sensory impairment, nurses should understand that ________ is an inaccurate statement. a. One of the major difficulties visually impaired parents experience is the skepticism of health care professionals. b. Visually impaired mothers cannot overcome the infants need for eye-to-eye contact. c. The best approach for the nurse is to assess the parents capabilities rather than focusing on their disabilities. d. Technologic advances, including the Internet, can provide deaf parents with a full range of parenting activities and information.

b. Visually impaired mothers cannot overcome the infants need for eye-to-eye contact.

Under which circumstance would it be unnecessary for the nurse to perform a vaginal examination? a. An admission to the hospital at the start of labor Test Bank - Maternal Child Nursing Care 5e (Perry, 2014) 261 b. When accelerations of the fetal heart rate (FHR) are noted c. On maternal perception of perineal pressure or the urge to bear down d. When membranes rupture

b. When accelerations of the fetal heart rate (FHR) are noted

A pregnant woman at 29 weeks of gestation has been diagnosed with preterm labor. Her labor is being controlled with tocolytic medications. She asks when she would be able to go home. Which response by the nurse is most accurate? a. After the baby is born. b. When we can stabilize your preterm labor and arrange home health visits. c. Whenever the doctor says that it is okay. d. It depends on what kind of insurance coverage you have.

b. When we can stabilize your preterm labor and arrange home health visits

With regard to dysfunctional labor, nurses should be aware that: a. Women who are underweight are more at risk. b. Women experiencing precipitous labor are about the only dysfunctionals not to be exhausted. c. Hypertonic uterine dysfunction is more common than hypotonic dysfunction. d. Abnormal labor patterns are most common in older women.

b. Women experiencing precipitous labor are about the only dysfunctionals not to be exhausted.

As relates to rubella and Rh issues, nurses should be aware that: a. Breastfeeding mothers cannot be vaccinated with the live attenuated rubella virus. b. Women should be warned that the rubella vaccination is teratogenic, and that they must avoid pregnancy for 1 month after vaccination. c. Rh immune globulin is safely administered intravenously because it cannot harm a nursing infant. d. Rh immune globulin boosts the immune system and thereby enhances the effectiveness of vaccinations.

b. Women should be warned that the rubella vaccination is teratogenic, and that they must avoid pregnancy for 1 month after vaccination.

Complications and risks associated with cesarean births include (Select all that apply): a. Placental abruption. b. Wound dehiscence. c. Hemorrhage. d. Urinary tract infections. e. Fetal injuries.

b. Wound dehiscence. c. Hemorrhage. d. Urinary tract infections. e. Fetal injuries.

According to demographic research, the woman least likely to breastfeed and therefore most likely to need education regarding the benefits and proper techniques of breastfeeding would be: a. A woman who is 30 to 35 years of age, Caucasian, and employed part time outside the home. b. A woman who is younger than 25 years of age, Hispanic, and unemployed. c. A woman who is younger than 25 years of age, African-American, and employed full time outside the home. d. A woman who is 35 years of age or older, Caucasian, and employed full time at home.

c. A woman who is younger than 25 years of age, African-American, and employed full time outside the home.

At a 2-month well-baby examination, it was discovered that a breastfed infant had only gained 10 ounces in the past 4 weeks. The mother and the nurse agree that, to gain weight faster, the infant needs to: a. Begin solid foods. b. Have a bottle of formula after every feeding. c. Add at least one extra breastfeeding session every 24 hours. d. Start iron supplements.

c. Add at least one extra breastfeeding session every 24 hours.

Of the many factors that influence parental responses, nurses should be conscious of negative stereotypes that apply to specific patient populations. Which response could be an inappropriate stereotype of adolescent mothers? a. An adolescent mothers egocentricity and unmet developmental needs interfere with her ability to parent effectively. b. An adolescent mother is likely to use less verbal instruction, be less responsive, and interact less positively than other mothers. c. Adolescent mothers have a higher documented incidence of child abuse. d. Mothers older than 35 often deal with more stress related to work and career issues and decreasing libido.

c. Adolescent mothers have a higher documented incidence of child abuse.

In caring for the mother who has abused (or is abusing) alcohol and for her infant, nurses should be aware that: a. The pattern of growth restriction of the fetus begun in prenatal life is halted after birth, and normal growth takes over. b. Two thirds of newborns with fetal alcohol syndrome (FAS) are boys. c. Alcohol-related neurodevelopmental disorders not sufficient to meet FAS criteria (learning disabilities, speech and language problems) are often not detected until the child goes to school. d. Both the distinctive facial features of the FAS infant and the diminished mental capacities tend toward normal over time.

c. Alcohol-related neurodevelopmental disorders not sufficient to meet FAS criteria (learning disabilities, speech and language problems) are often not detected until the child goes to school.

A woman in labor is breathing into a mouthpiece just before the start of her regular contractions. As she inhales, a valve opens, and gas is released. She continues to inhale the gas slowly and deeply until the contraction starts to subside. When the inhalation stops, the valve closes. This procedure is: a. Not used much anymore. b. Likely to be used in the second stage of labor but not in the first stage. c. An application of nitrous oxide. d. A prelude to cesarean birth.

c. An application of nitrous oxide.

A woman is having her first child. She has been in labor for 15 hours. Two hours ago her vaginal examination revealed the cervix to be dilated to 5 cm and 100% effaced, and the presenting part was at station 0. Five minutes ago her vaginal examination indicated that there had been no change. What abnormal labor pattern is associated with this description? a. Prolonged latent phase b. Protracted active phase c. Arrest of active phase d. Protracted descent

c. Arrest of active phase

The best way for the nurse to promote and support the maternal-infant bonding process is to: a. Help the mother identify her positive feelings toward the newborn. b. Encourage the mother to provide all newborn care. c. Assist the family with rooming-in. d. Return the newborn to the nursery during sleep periods.

c. Assist the family with rooming-in.

An infant boy was born just a few minutes ago. The nurse is conducting the initial assessment. Part of the assessment includes the Apgar score. The Apgar assessment is performed: a. Only if the newborn is in obvious distress. b. Once by the obstetrician, just after the birth. c. At least twice, 1 minute and 5 minutes after birth. d. Every 15 minutes during the newborns first hour after birth.

c. At least twice, 1 minute and 5 minutes after birth.

The nurse administers vitamin K to the newborn for which reason? a. Most mothers have a diet deficient in vitamin K, which results in the infants being deficient. b. Vitamin K prevents the synthesis of prothrombin in the liver and must be given by injection. c. Bacteria that synthesize vitamin K are not present in the newborns intestinal tract. d. The supply of vitamin K is inadequate for at least 3 to 4 months, and the newborn must be supplemented.

c. Bacteria that synthesize vitamin K are not present in the newborns intestinal tract.

According to standard professional thinking, nurses should auscultate the fetal heart rate (FHR): a. Every 15 minutes in the active phase of the first stage of labor in the absence of risk factors. b. Every 20 minutes in the second stage, regardless of whether risk factors are present. c. Before and after ambulation and rupture of membranes. d. More often in a woman's first pregnancy.

c. Before and after ambulation and rupture of membranes.

While discussing the societal impacts of breastfeeding, the nurse should be cognizant of the benefits and educate the patient accordingly. Which statement as part of this discussion would be incorrect? a. Breastfeeding requires fewer supplies and less cumbersome equipment. b. Breastfeeding saves families money. c. Breastfeeding costs employers in terms of time lost from work. d. Breastfeeding benefits the environment.

c. Breastfeeding costs employers in terms of time lost from work.

With regard to the nutrient needs of breastfed and formula-fed infants, nurses should be understand that: a. Breastfed infants need extra water in hot climates. b. During the first 3 months breastfed infants consume more energy than do formula-fed infants. c. Breastfeeding infants should receive oral vitamin D drops daily at least during the first 2 months. d. Vitamin K injections at birth are not needed for infants fed on specially enriched formula.

c. Breastfeeding infants should receive oral vitamin D drops daily at least during the first 2 months.

A breastfeeding woman develops engorged breasts at 3 days postpartum. What action would help this woman achieve her goal of reducing the engorgement? The woman: a. Skips feedings to let her sore breasts rest. b. Avoids using a breast pump. c. Breastfeeds her infant every 2 hours. d. Reduces her fluid intake for 24 hours.

c. Breastfeeds her infant every 2 hours.

A new mother wants to be sure that she is meeting her daughters needs while feeding her commercially prepared infant formula. The nurse should evaluate the mothers knowledge about appropriate infant care. The mother meets her child's needs when she: a. Adds rice cereal to her formula at 2 weeks of age to ensure adequate nutrition. b. Warms the bottles using a microwave oven. c. Burps her infant during and after the feeding as needed. d. Refrigerates any leftover formula for the next feeding.

c. Burps her infant during and after the feeding as needed.

The nurse hears a primiparous woman talking to her son and telling him that his chin is just like his dad's chin. This woman's statement reflects: a. Mutuality. b. Synchrony. c. Claiming. d. Reciprocity.

c. Claiming.

Near the end of the first week of life, an infant who has not been treated for any infection develops a copper-colored, maculopapular rash on the palms and around the mouth and anus. The newborn is showing signs of: a. Gonorrhea. b. Herpes simplex virus infection. c. Congenital syphilis. d. Human immunodeficiency virus.

c. Congenital syphilis.

The nurse teaches a pregnant woman about the characteristics of true labor contractions. The nurse evaluates the woman's understanding of the instructions when she states, True labor contractions will: a. Subside when I walk around. b. Cause discomfort over the top of my uterus. c. Continue and get stronger even if I relax and take a shower. d. Remain irregular but become stronger.

c. Continue and get stronger even if I relax and take a shower.

Which description of the phases of the second stage of labor is accurate? a. Latent phase: Feeling sleepy, fetal station 2+ to 4+, duration 30 to 45 minutes b. Active phase: Overwhelmingly strong contractions, Ferguson reflux activated, duration 5 to 15 minutes c. Descent phase: Significant increase in contractions, Ferguson reflux activated, average duration varied d. Transitional phase: Woman laboring down, fetal station 0, duration 15 minutes

c. Descent phase: Significant increase in contractions, Ferguson reflux activated, average duration varied

If a woman is at risk for thrombus and is not ready to ambulate, nurses may intervene by performing a number of interventions. Which intervention should the nurse avoid? a. Putting the patient in antiembolic stockings (TED hose) and/or sequential compression device (SCD) boots. b. Having the patient flex, extend, and rotate her feet, ankles, and legs. c. Having the patient sit in a chair. d. Notifying the physician immediately if a positive Homans sign occurs.

c. Having the patient sit in a chair.

Despite popular belief, there is a rare type of hemophilia that affects women of childbearing age. von Willebrand disease is the most common of the hereditary bleeding disorders and can affect males and females alike. It results from a factor VIII deficiency and platelet dysfunction. Although factor VIII levels increase naturally during pregnancy, there is an increased risk for postpartum hemorrhage from birth until 4 weeks after delivery as levels of von Willebrand factor (vWf) and factor VIII decrease. The treatment that should be considered first for the client with von Willebrand disease who experiences a postpartum hemorrhage is: a. Cryoprecipitate. b. Factor VIII and vWf. c. Desmopressin. d. Hemabate.

c. Desmopressin.

The maternity nurse understands that as the uterus contracts during labor, maternal-fetal exchange of oxygen and waste products: a. Continues except when placental functions are reduced. b. Increases as blood pressure decreases. c. Diminishes as the spiral arteries are compressed. d. Is not significantly affected.

c. Diminishes as the spiral arteries are compressed.

The least common cause of long, difficult, or abnormal labor (dystocia) is: a. Midplane contracture of the pelvis. b. Compromised bearing-down efforts as a result of pain medication. c. Disproportion of the pelvis. d. Low-lying placenta.

c. Disproportion of the pelvis.

While assessing the integument of a 24-hour-old newborn, the nurse notes a pink, papular rash with vesicles superimposed on the thorax, back, and abdomen. The nurse should: a. Notify the physician immediately. b. Move the newborn to an isolation nursery. c. Document the finding as erythema toxicum. d. Take the newborns temperature and obtain a culture of one of the vesicles.

c. Document the finding as erythema toxicum.

The uterine contractions of a woman early in the active phase of labor are assessed by an internal uterine pressure catheter (IUPC). The nurse notes that the intrauterine pressure at the peak of the contraction ranges from 65 to 70 mm Hg and the resting tone range is 6 to 10 mm Hg. The uterine contractions occur every 3 to 4 minutes and last an average of 55 to 60 seconds. On the basis of this information, the nurse should: a. Notify the woman's primary health care provider immediately. b. Prepare to administer an oxytocic to stimulate uterine activity. c. Document the findings because they reflect the expected contraction pattern for the active phase of labor. d. Prepare the woman for the onset of the second stage of labor.

c. Document the findings because they reflect the expected contraction pattern for the active phase of labor.

A primigravida has just delivered a healthy infant girl. The nurse is about to administer erythromycin ointment in the infants eyes when the mother asks, What is that medicine for? The nurse responds: a. It is an eye ointment to help your baby see you better. b. It is to protect your baby from contracting herpes from your vaginal tract. c. Erythromycin is given prophylactically to prevent a gonorrheal infection. d. This medicine will protect your baby's eyes from drying out over the next few days.

c. Erythromycin is given prophylactically to prevent a gonorrheal infection.

Which action is correct when palpation is used to assess the characteristics and pattern of uterine contractions? a. Place the hand on the abdomen below the umbilicus and palpate uterine tone with the fingertips. b. Determine the frequency by timing from the end of one contraction to the end of the next contraction. c. Evaluate the intensity by pressing the fingertips into the uterine fundus. d. Assess uterine contractions every 30 minutes throughout the first stage of labor.

c. Evaluate the intensity by pressing the fingertips into the uterine fundus.

With regard to parents early and extended contact with their infant and the relationships built, nurses should be aware that: a. Immediate contact is essential for the parent-child relationship. b. Skin-to-skin contact is preferable to contact with the body totally wrapped in a blanket. c. Extended contact is especially important for adolescents and low-income women because they are at risk for parenting inadequacies. d. Mothers need to take precedence over their partners and other family matters.

c. Extended contact is especially important for adolescents and low-income women because they are at risk for parenting inadequacies.

In the classification of newborns by gestational age and birth weight, the appropriate for gestational age (AGA) weight would: a. Fall between the 25th and 75th percentiles for the infants age. b. Depend on the infants length and the size of the head. c. Fall between the 10th and 90th percentiles for the infants age. d. Be modified to consider intrauterine growth restriction (IUGR).

c. Fall between the 10th and 90th percentiles for the infants age.

A nulliparous woman who has just begun the second stage of her labor would most likely: a. Experience a strong urge to bear down. b. Show perineal bulging. c. Feel tired yet relieved that the worst is over. d. Show an increase in bright red bloody show.

c. Feel tired yet relieved that the worst is over.

As relates to the condition and reconditioning of the urinary system after childbirth, nurses should be aware that: a. Kidney function returns to normal a few days after birth. b. Diastasis recti abdominis is a common condition that alters the voiding reflex. c. Fluid loss through perspiration and increased urinary output accounts for a weight loss of more than 2 kg during the puerperium. d. With adequate emptying of the bladder, bladder tone usually is restored 2 to 3 weeks after childbirth.

c. Fluid loss through perspiration and increased urinary output accounts for a weight loss of more than 2 kg during the puerperium.

While providing care to a patient in active labor, the nurse should instruct the woman that: a. The supine position commonly used in the United States increases blood flow. b. The all fours position, on her hands and knees, is hard on her back. c. Frequent changes in position will help relieve her fatigue and increase her comfort. d. In a sitting or squatting position, her abdominal muscles will have to work harder.

c. Frequent changes in position will help relieve her fatigue and increase her comfort.

Which documentation on a womans chart on postpartum day 14 indicates a normal involution process? a. Moderate bright red lochial flow b. Breasts firm and tender c. Fundus below the symphysis and not palpable d. Episiotomy slightly red and puffy

c. Fundus below the symphysis and not palpable

On observing a woman on her first postpartum day sitting in bed while her newborn lies awake in the bassinet, the nurse should: a. Realize that this situation is perfectly acceptable. b. Offer to hand the baby to the woman. c. Hand the baby to the woman. d. Explain taking in to the woman.

c. Hand the baby to the woman.

Because the risk for childbirth complications may be revealed, nurses should know that the point of maximal intensity (PMI) of the fetal heart tone (FHT) is: a. Usually directly over the fetal abdomen. b. In a vertex position heard above the mothers umbilicus. c. Heard lower and closer to the midline of the mothers abdomen as the fetus descends and rotates internally. d. In a breech position heard below the mothers umbilicus.

c. Heard lower and closer to the midline of the mothers abdomen as the fetus descends and rotates internally.

The nurse practicing in the perinatal setting should promote kangaroo care regardless of an infants gestational age. This intervention: a. Is adopted from classical British nursing traditions. b. Helps infants with motor and central nervous system impairment. c. Helps infants to interact directly with their parents and enhances their temperature regulation. d. Gets infants ready for breastfeeding.

c. Helps infants to interact directly with their parents and enhances their temperature regulation.

Nurses with an understanding of cultural differences regarding likely reactions to pain may be better able to help clients. Nurses should know that _____ women may be stoic until late in labor, when they may become vocal and request pain relief. a. Chinese b. Arab or Middle Eastern c. Hispanic d. African-American

c. Hispanic

An infant was born 2 hours ago at 37 weeks of gestation and weighing 4.1 kg. The infant appears chubby with a flushed complexion and is very tremulous. The tremors are most likely the result of: a. Birth injury. b. Hypocalcemia. c. Hypoglycemia. d. Seizures.

c. Hypoglycemia.

As related to laboratory tests and diagnostic tests in the hospital after birth, nurses should be aware that: a. All states test for phenylketonuria (PKU), hypothyroidism, cystic fibrosis, and sickle cell diseases. b. Federal law prohibits newborn genetic testing without parental consent. c. If genetic screening is done before the infant is 24 hours old, it should be repeated at age 1 to 2 weeks. d. Hearing screening is now mandated by federal law.

c. If genetic screening is done before the infant is 24 hours old, it should be repeated at age 1 to 2 weeks.

The laboring woman who imagines her body opening to let the baby out is using a mental technique called: a. Dissociation. b. Effleurage. c. Imagery. d. Distraction.

c. Imagery.

As the nurse assists a new mother with breastfeeding, the client asks, If formula is prepared to meet the nutritional needs of the newborn, what is in breast milk that makes it better? The nurses best response is that it contains: a. More calories. b. Essential amino acids. c. Important immunoglobulins. d. More calcium.

c. Important immunoglobulins.

As related to central nervous system injuries that could occur to the infant during labor and birth, nurses should be aware that: a. Intracranial hemorrhage (ICH) as a result of birth trauma is more likely to occur in the preterm, low-birth-weight infant. b. Subarachnoid hemorrhage (the most common form of ICH) occurs in term infants as a result of hypoxia. c. In many infants signs of hemorrhage in a full-term infant are absent and are diagnosed only through laboratory tests. d. Spinal cord injuries almost always result from forceps-assisted deliveries.

c. In many infants signs of hemorrhage in a full-term infant are absent and are diagnosed only through laboratory tests.

Complicated bereavement: a. Occurs when, in multiple births, one child dies, and the other or others live. b. Is a state in which the parents are ambivalent, as with an abortion. c. Is an extremely intense grief reaction that persists for a long time. d. Is felt by the family of adolescent mothers who lose their babies.

c. Is an extremely intense grief reaction that persists for a long time.

The nurse providing care to a woman in labor should understand that cesarean birth: a. Is declining in frequency in the twenty-first century in the United States. b. Is more likely to be performed for poor women in public hospitals who do not receive the nurse counseling as do wealthier clients. c. Is performed primarily for the benefit of the fetus. d. Can be either elected or refused by women as their absolute legal right.

c. Is performed primarily for the benefit of the fetus.

As relates to the use of tocolytic therapy to suppress uterine activity, nurses should be aware that: a. The drugs can be given efficaciously up to the designated beginning of term at 37 weeks. b. There are no important maternal (as opposed to fetal) contraindications. c. Its most important function is to afford the opportunity to administer antenatal glucocorticoids. d. If the client develops pulmonary edema while receiving tocolytics, intravenous (IV) fluids should be given.

c. Its most important function is to afford the opportunity to administer antenatal glucocorticoids.

Two days ago a woman gave birth to a full-term infant. Last night she awakened several times to urinate and noted that her gown and bedding were wet from profuse diaphoresis. One mechanism for the diaphoresis and diuresis that this woman is experiencing during the early postpartum period is: a. Elevated temperature caused by postpartum infection. b. Increased basal metabolic rate after giving birth. c. Loss of increased blood volume associated with pregnancy. d. Increased venous pressure in the lower extremities.

c. Loss of increased blood volume associated with pregnancy.

During the initial acute distress phase of grieving, parents still must make unexpected and unwanted decisions about funeral arrangements and even naming the baby. The nurses role should be to: a. Take over as much as possible to relieve the pressure. b. Encourage grandparents to take over. c. Make sure the parents themselves approve the final decisions. d. Let them alone to work things out.

c. Make sure the parents themselves approve the final decisions.

What infection is contracted mostly by first-time mothers who are breastfeeding? a. Endometritis b. Wound infections c. Mastitis d. Urinary tract infections

c. Mastitis

The nurse is performing a gestational age and physical assessment on the newborn. The infant appears to have an excessive amount of saliva. The nurse recognizes that this finding: a. Is normal. b. Indicates that the infant is hungry. c. May indicate that the infant has a tracheoesophageal fistula or esophageal atresia. d. May indicate that the infant has a diaphragmatic hernia.

c. May indicate that the infant has a tracheoesophageal fistula or esophageal atresia.

The nurse caring for the woman in labor should understand that increased variability of the fetal heart rate may be caused by: a. Narcotics. b. Barbiturates. c. Methamphetamines. d. Tranquilizers.

c. Methamphetamines.

As a result of large body surface in relation to weight, the preterm infant is at high risk for heat loss and cold stress. By understanding the four mechanisms of heat transfer (convection, conduction, radiation, and evaporation), the nurse can create an environment for the infant that prevents temperature instability. While evaluating the plan that has been implemented, the nurse knows that the infant is experiencing cold stress when he or she exhibits: a. Decreased respiratory rate. b. Bradycardia followed by an increased heart rate. c. Mottled skin with acrocyanosis. d. Increased physical activity.

c. Mottled skin with acrocyanosis.

A laboring woman received an opioid agonist (meperidine) intravenously 90 minutes before she gave birth. Which medication should be available to reduce the postnatal effects of Demerol on the neonate? a. Fentanyl (Sublimaze) b. Promethazine (Phenergan) c. Naloxone (Narcan) d. Nalbuphine (Nubain)

c. Naloxone (Narcan)

Providing care for the neonate born to a mother who abuses substances can present a challenge for the health care team. Nursing care for this infant requires a multisystem approach. The first step in the provision of this care is: a. Pharmacologic treatment. b. Reduction of environmental stimuli. c. Neonatal abstinence syndrome scoring. d. Adequate nutrition and maintenance of fluid and electrolyte balance.

c. Neonatal abstinence syndrome scoring.

In administering vitamin K to the infant shortly after birth, the nurse understands that vitamin K is: a. Important in the production of red blood cells. b. Necessary in the production of platelets. c. Not initially synthesized because of a sterile bowel at birth. d. Responsible for the breakdown of bilirubin and prevention of jaundice.

c. Not initially synthesized because of a sterile bowel at birth.

The early postpartum period is a time of emotional and physical vulnerability. Many mothers can easily become psychologically overwhelmed by the reality of their new parental responsibilities. Fatigue compounds these issues. Although the baby blues are a common occurrence in the postpartum period, about one-half million women in America experience a more severe syndrome known as postpartum depression (PPD). Which statement regarding PPD is essential for the nurse to be aware of when attempting to formulate a nursing diagnosis? a. PPD symptoms are consistently severe. b. This syndrome affects only new mothers. c. PPD can easily go undetected. d. Only mental health professionals should teach new parents about this condition.

c. PPD can easily go undetected.

The first and most important nursing intervention when a nurse observes profuse postpartum bleeding is to: a. Call the woman's primary health care provider. b. Administer the standing order for an oxytocic. c. Palpate the uterus and massage it if it is boggy. d. Assess maternal blood pressure and pulse for signs of hypovolemic shock.

c. Palpate the uterus and massage it if it is boggy.

As related to the eventual discharge of the high risk newborn or transfer to a different facility, nurses and families should be aware that: a. Infants will stay in the neonatal intensive care unit (NICU) until they are ready to go home. b. Once discharged to home, the high risk infant should be treated like any healthy term newborn. c. Parents of high risk infants need special support and detailed contact information. d. If a high risk infant and mother need transfer

c. Parents of high risk infants need special support and detailed contact information.

A meconium stool can be differentiated from a transitional stool in the newborn because the meconium stool is: a. Seen at age 3 days. b. The residue of a milk curd. c. Passed in the first 12 hours of life. d. Lighter in color and looser in consistency.

c. Passed in the first 12 hours of life.

What marks on a babys skin may indicate an underlying problem that requires notification of a physician? a. Mongolian spots on the back b. Telangiectatic nevi on the nose or nape of the neck c. Petechiae scattered over the infants body d. Erythema toxicum anywhere on the body

c. Petechiae scattered over the infants body

A newborn is jaundiced and receiving phototherapy via ultraviolet bank lights. An appropriate nursing intervention when caring for an infant with hyperbilirubinemia and receiving phototherapy by this method would be to: a. Apply an oil-based lotion to the newborns skin to prevent dying and cracking. b. Limit the newborns intake of milk to prevent nausea, vomiting, and diarrhea. c. Place eye shields over the newborns closed eyes. d. Change the newborns position every 4 hours.

c. Place eye shields over the newborns closed eyes.

To initiate the milk ejection reflex (MER), the mother should be advised to: a. Wear a firm-fitting bra. b. Drink plenty of fluids. c. Place the infant to the breast. d. Apply cool packs to her breast.

c. Place the infant to the breast.

The baseline fetal heart rate (FHR) is the average rate during a 10-minute segment. Changes in FHR are categorized as periodic or episodic. These patterns include both accelerations and decelerations. The labor nurse is evaluating the patients most recent 10-minute segment on the monitor strip and notes a late deceleration. This is likely to be caused by which physiologic alteration (Select all that apply)? a. Spontaneous fetal movement b. Compression of the fetal head c. Placental abruption d. Cord around the baby's neck e. Maternal supine hypotension

c. Placental abruption e. Maternal supine hypotension

During a phone follow-up conversation with a woman who is 4 days postpartum, the woman tells the nurse, I don't know what's wrong. I love my son, but I feel so let down. I seem to cry for no reason! The nurse would recognize that the woman is experiencing: a. Taking-in. b. Postpartum depression (PPD). c. Postpartum (PP) blues. d. Attachment difficulty.

c. Postpartum depression (PPD).

Postpartal overdistention of the bladder and urinary retention can lead to which complications? a. Postpartum hemorrhage and eclampsia b. Fever and increased blood pressure c. Postpartum hemorrhage and urinary tract infection d. Urinary tract infection and uterine rupture

c. Postpartum hemorrhage and urinary tract infection

For clinical purposes, preterm and post-term infants are defined as: a. Preterm before 34 weeks if appropriate for gestational age (AGA) and before 37 weeks if small for gestational age (SGA). b. Post-term after 40 weeks if large for gestational age (LGA) and beyond 42 weeks if AGA. c. Preterm before 37 weeks, and post-term beyond 42 weeks, no matter the size for gestational age at birth. d. Preterm, SGA before 38 to 40 weeks, and post-term, LGA beyond 40 to 42 weeks.

c. Preterm before 37 weeks, and post-term beyond 42 weeks, no matter the size for gestational age at birth.

In planning for home care of a woman with preterm labor, which concern must the nurse address? a. Nursing assessments will be different from those done in the hospital setting. b. Restricted activity and medications will be necessary to prevent recurrence of preterm labor. c. Prolonged bed rest may cause negative physiologic effects. d. Home health care providers will be necessary.

c. Prolonged bed rest may cause negative physiologic effects.

Fetal bradycardia is most common during: a. Intraamniotic infection. b. Fetal anemia. c. Prolonged umbilical cord compression. d. Tocolytic treatment using terbutaline.

c. Prolonged umbilical cord compression.

When assessing the fetus using Leopold maneuvers, the nurse feels a round, firm, movable fetal part in the fundal portion of the uterus and a long, smooth surface in the mothers right side close to midline. What is the likely position of the fetus? a. ROA b. LSP c. RSA d. LOA

c. RSA

Which type of formula is not diluted before being administered to an infant? a. Powdered b. Concentrated c. Ready-to-use d. Modified cows milk

c. Ready-to-use

If the patients white blood cell (WBC) count is 25,000/mm on her second postpartum day, the nurse should: a. Tell the physician immediately. b. Have the laboratory draw blood for reanalysis. c. Recognize that this is an acceptable range at this point postpartum. d. Begin antibiotic therapy immediately.

c. Recognize that this is an acceptable range at this point postpartum.

The nurse thoroughly dries the infant immediately after birth primarily to: a. Stimulate crying and lung expansion. b. Remove maternal blood from the skin surface. c. Reduce heat loss from evaporation. d. Increase blood supply to the hands and feet.

c. Reduce heat loss from evaporation.

With regard to the gastrointestinal (GI) system of the newborn, nurses should be aware that: a. The newborns cheeks are full because of normal fluid retention. b. The nipple of the bottle or breast must be placed well inside the baby's mouth because teeth have been developing in utero, and one or more may even be through. c. Regurgitation during the first day or two can be reduced by burping the infant and slightly elevating the baby's head. d. Bacteria are already present in the infants GI tract at birth because they traveled through the placenta.

c. Regurgitation during the first day or two can be reduced by burping the infant and slightly elevating the baby's head.

Infants of mothers with diabetes (IDMs) are at higher risk for developing: a. Anemia. b. Hyponatremia. c. Respiratory distress syndrome. d. Sepsis.

c. Respiratory distress syndrome.

Prepidil (prostaglandin gel) has been ordered for a pregnant woman at 43 weeks of gestation. The nurse recognizes that this medication will be administered to: a. Enhance uteroplacental perfusion in an aging placenta. b. Increase amniotic fluid volume. c. Ripen the cervix in preparation for labor induction. d. Stimulate the amniotic membranes to rupture.

c. Ripen the cervix in preparation for labor induction.

A primigravida at 40 weeks of gestation is having uterine contractions every 1.5 to 2 minutes and says that they are very painful. Her cervix is dilated 2 cm and has not changed in 3 hours. The woman is crying and wants an epidural. What is the likely status of this womans labor? a. She is exhibiting hypotonic uterine dysfunction. b. She is experiencing a normal latent stage. c. She is exhibiting hypertonic uterine dysfunction. d. She is experiencing pelvic dystocia.

c. She is exhibiting hypertonic uterine dysfunction.

With regard to basic care of the breastfeeding mother, nurses should be able to advise her that she: a. Will need an extra 1000 calories a day to maintain energy and produce milk. b. Can go back to prepregnancy consumption patterns of any drinks, as long as she ingests enough calcium. c. Should avoid trying to lose large amounts of weight. d. Must avoid exercising because it is too fatiguing.

c. Should avoid trying to lose large amounts of weight.

An infant is to receive gastrostomy feedings. What intervention should the nurse institute to prevent bloating, gastrointestinal reflux into the esophagus, vomiting, and respiratory compromise? a. Rapid bolusing of the entire amount in 15 minutes b. Warm cloths to the abdomen for the first 10 minutes c. Slow, small, warm bolus feedings over 30 minutes d. Cold, medium bolus feedings over 20 minutes

c. Slow, small, warm bolus feedings over 30 minutes

The normal term infant has little difficulty clearing the airway after birth. Most secretions are brought up to the oropharynx by the cough reflex. However, if the infant has excess secretions, the mouth and nasal passages can be cleared easily with a bulb syringe. When instructing parents on the correct use of this piece of equipment, it is important that the nurse teach them to: a. Avoid suctioning the nares. b. Insert the compressed bulb into the center of the mouth. c. Suction the mouth first. d. Remove the bulb syringe from the crib when finished.

c. Suction the mouth first.

A plan of care for an infant experiencing symptoms of drug withdrawal should include: a. Administering chloral hydrate for sedation. b. Feeding every 4 to 6 hours to allow extra rest. c. Swaddling the infant snugly and holding the baby tightly. d. Playing soft music during feeding.

c. Swaddling the infant snugly and holding the baby tightly.

A first-time dad is concerned that his 3-day-old daughters skin looks yellow. In the nurses explanation of physiologic jaundice, what fact should be included? a. Physiologic jaundice occurs during the first 24 hours of life. b. Physiologic jaundice is caused by blood incompatibilities between the mother and infant blood types. c. The bilirubin levels of physiologic jaundice peak between the second and fourth days of life. d. This condition is also known as breast milk jaundice.

c. The bilirubin levels of physiologic jaundice peak between the second and fourth days of life.

The nurse recognizes that a woman is in true labor when she states: a. I passed some thick, pink mucus when I urinated this morning. b. My bag of waters just broke. c. The contractions in my uterus are getting stronger and closer together. d. My baby dropped, and I have to urinate more frequently now.

c. The contractions in my uterus are getting stronger and closer together.

In documenting labor experiences, nurses should know that a uterine contraction is described according to all these characteristics except: a. Frequency (how often contractions occur). b. Intensity (the strength of the contraction at its peak). c. Resting tone (the tension in the uterine muscle). d. Appearance (shape and height).

d. Appearance (shape and height).

With regard to the turns and other adjustments of the fetus during the birth process, known as the mechanism of labor, nurses should be aware that: a. The seven critical movements must progress in a more or less orderly sequence. b. Asynclitism sometimes is achieved by means of the Leopold maneuver. c. The effects of the forces determining descent are modified by the shape of the woman's pelvis and the size of the fetal head. d. At birth the baby is said to achieve restitution (i.e., a return to the C-shape of the womb).

c. The effects of the forces determining descent are modified by the shape of the woman's pelvis and the size of the fetal head.

With regard to factors that affect how the fetus moves through the birth canal, nurses should be aware that: a. The fetal attitude describes the angle at which the fetus exits the uterus. b. Of the two primary fetal lies, the horizontal lie is that in which the long axis of the fetus is parallel to the long axis of the mother. c. The normal attitude of the fetus is called general flexion. d. The transverse lie is preferred for vaginal birth.

c. The normal attitude of the fetus is called general flexion.

A mother with mastitis is concerned about breastfeeding while she has an active infection. The nurse should explain that: a. The infant is protected from infection by immunoglobulins in the breast milk. b. The infant is not susceptible to the organisms that cause mastitis. c. The organisms that cause mastitis are not passed to the milk. d. The organisms will be inactivated by gastric acid.

c. The organisms that cause mastitis are not passed to the milk.

With regard to the newborns developing cardiovascular system, nurses should be aware that: a. The heart rate of a crying infant may rise to 120 beats/min. b. Heart murmurs heard after the first few hours are cause for concern. c. The point of maximal impulse (PMI) often is visible on the chest wall. d. Persistent bradycardia may indicate respiratory distress syndrome (RDS).

c. The point of maximal impulse (PMI) often is visible on the chest wall.

The ultrasound transducer can accurately measure short-term variability and beat-to-beat changes in the fetal heart rate. b. The tocotransducer can measure and record the frequency, regularity, intensity, and approximate duration of uterine contractions (UCs). c. The tocotransducer is especially valuable for measuring uterine activity during the first stage of labor. d. Once correctly applied by the nurse, the transducer need not be repositioned even when the woman changes positions.

c. The tocotransducer is especially valuable for measuring uterine activity during the first stage of labor.

Which nursing assessment indicates that a woman who is in second-stage labor is almost ready to give birth? a. The fetal head is felt at 0 station during vaginal examination. b. Bloody mucus discharge increases. c. The vulva bulges and encircles the fetal head. d. The membranes rupture during a contraction.

c. The vulva bulges and encircles the fetal head.

The nurse knows that the second stage of labor, the descent phase, has begun when: a. The amniotic membranes rupture. b. The cervix cannot be felt during a vaginal examination. c. The woman experiences a strong urge to bear down. d. The presenting part is below the ischial spines.

c. The woman experiences a strong urge to bear down.

A woman who has recently given birth complains of pain and tenderness in her leg. On physical examination the nurse notices warmth and redness over an enlarged, hardened area. The nurse should suspect __________ and should confirm the diagnosis by ___________. a. Disseminated intravascular coagulation; asking for laboratory tests b. von Willebrand disease; noting whether bleeding times have been extended c. Thrombophlebitis; using real-time and color Doppler ultrasound d. Coagulopathies; drawing blood for laboratory analysis

c. Thrombophlebitis; using real-time and color Doppler ultrasound

As part of Standard Precautions, nurses wear gloves when handling the newborn. The chief reason is: a. To protect the baby from infection. b. That it is part of the Apgar protocol. c. To protect the nurse from contamination by the newborn. d. the nurse has primary responsibility for the baby during the first 2 hours.

c. To protect the nurse from contamination by the newborn.

Surgical, medical, or mechanical methods may be used for labor induction. Which technique is considered a mechanical method of induction? a. Amniotomy b. Intravenous Pitocin c. Transcervical catheter d. Vaginal insertion of prostaglandins

c. Transcervical catheter

The nurse caring for a woman in labor understands that prolonged decelerations: a. Are a continuing pattern of benign decelerations that do not require intervention. b. Constitute a baseline change when they last longer than 5 minutes. c. Usually are isolated events that end spontaneously. d. Require the usual fetal monitoring by the nurse.

c. Usually are isolated events that end spontaneously.

A 25-year-old gravida 2, para 2-0-0-2 gave birth 4 hours ago to a 9-pound, 7-ounce boy after augmentation of labor with Pitocin. She puts on her call light and asks for her nurse right away, stating, I'm bleeding a lot. The most likely cause of postpartum hemorrhage in this woman is: a. Retained placental fragments. b. Unrepaired vaginal lacerations. c. Uterine atony. d. Puerperal infection.

c. Uterine atony.

The nurse caring for the woman in labor should understand that maternal hypotension can result in: a. Early decelerations. b. Fetal dysrhythmias. c. Uteroplacental insufficiency. d. Spontaneous rupture of membranes.

c. Uteroplacental insufficiency.

The nurse providing care for the laboring woman should understand that late fetal heart rate (FHR) decelerations are the result of: a. Altered cerebral blood flow. c. Uteroplacental insufficiency. b. Umbilical cord compression. d. Meconium fluid.

c. Uteroplacental insufficiency.

Nursing activities that promote parent-infant attachment are many and varied. One activity that should not be overlooked is management of the environment. While providing routine mother-baby care, the nurse should ensure that: a. The baby is able to return to the nursery at night so that the new mother can sleep. b. Routine times for care are established to reassure the parents. c. The father should be encouraged to go home at night to prepare for mother-baby discharge. d. An environment that fosters as much privacy as possible should be created.

d. An environment that fosters as much privacy as possible should be created.

A woman experienced a miscarriage at 10 weeks of gestation and had a dilation and curettage (D&C). She states that she is just fine and wants to go home as soon as possible. While you are assessing her responses to her loss, she tells you that she had purchased some baby things and had picked out a name. On the basis of your assessment of her responses, what nursing intervention would you use first? a. Ready her for discharge. b. Notify pastoral care to offer her a blessing. c. Ask her whether she would like to see what was obtained from her D&C. d. Ask her what name she had picked out for her baby.

d. Ask her what name she had picked out for her baby.

When a nulliparous woman telephones the hospital to report that she is in labor, the nurse initially should: a. Tell the woman to stay home until her membranes rupture. b. Emphasize that food and fluid intake should stop. c. Arrange for the woman to come to the hospital for labor evaluation. d. Ask the woman to describe why she believes she is in labor.

d. Ask the woman to describe why she believes she is in labor.

The obstetric nurse is preparing the patient for an emergency cesarean birth, with no time to administer spinal anesthesia. The nurse is aware and prepared for the greatest risk of administering general anesthesia to the patient. This risk is: a. Respiratory depression. b. Uterine relaxation. c. Inadequate muscle relaxation. d. Aspiration of stomach contents.

d. Aspiration of stomach contents.

A multiparous woman has been in labor for 8 hours. Her membranes have just ruptured. The nurses initial response would be to: a. Prepare the woman for imminent birth. b. Notify the woman's primary health care provider. c. Document the characteristics of the fluid. d. Assess the fetal heart rate and pattern.

d. Assess the fetal heart rate and pattern.

The priority nursing intervention after an amniotomy should be to: a. Assess the color of the amniotic fluid. b. Change the patients gown. c. Estimate the amount of amniotic fluid. d. Assess the fetal heart rate.

d. Assess the fetal heart rate.

Which nursing action is most appropriate to correct a boggy uterus that is displaced above and to the right of the umbilicus? a. Notify the physician of an impending hemorrhage. b. Assess the blood pressure and pulse. c. Evaluate the lochia. d. Assist the patient in emptying her bladder.

d. Assist the patient in emptying her bladder.

After they are born, a crying infant may be soothed by being held in a position in which the newborn can hear the mothers heartbeat. This phenomenon is known as: a. Entrainment. b. Reciprocity. c. Synchrony. d. Biorhythmicity.

d. Biorhythmicity.

Which method of pain management is safest for a gravida 3 para 2 admitted at 8 cm cervical dilation? a. Epidural anesthesia b. Narcotics c. Spinal block d. Breathing and relaxation techniques

d. Breathing and relaxation techniques

Premature infants who exhibit 5 to 10 seconds of respiratory pauses followed by 10 to 15 seconds of compensatory rapid respiration are: a. Suffering from sleep or wakeful apnea. b. Experiencing severe swings in blood pressure. c. Trying to maintain a neutral thermal environment. d. Breathing in a respiratory pattern common to premature infants.

d. Breathing in a respiratory pattern common to premature infants.

When assessing the relative advantages and disadvantages of internal and external electronic fetal monitoring, nurses comprehend that both: a. Can be used when membranes are intact. b. Measure the frequency, duration, and intensity of uterine contractions. c. May need to rely on the woman to indicate when uterine activity (UA) is occurring. d. Can be used during the antepartum and intrapartum periods.

d. Can be used during the antepartum and intrapartum periods.

One reason the brain is vulnerable to nutritional deficiencies and trauma in early infancy is the: a. Incompletely developed neuromuscular system. b. Primitive reflex system. c. Presence of various sleep-wake states. d. Cerebellum growth spurt.

d. Cerebellum growth spurt.

The nurse caring for the postpartum woman understands that breast engorgement is caused by: a. Overproduction of colostrum. b. Accumulation of milk in the lactiferous ducts and glands. c. Hyperplasia of mammary tissue. d. Congestion of veins and lymphatics.

d. Congestion of veins and lymphatics.

The nurse caring for the postpartum woman understands that breast engorgement is caused by: a. Overproduction of colostrum. b. Accumulation of milk in the lactiferous ducts. c. Hyperplasia of mammary tissue. d. Congestion of veins and lymphatics.

d. Congestion of veins and lymphatics.

If nonsurgical treatment for late postpartum hemorrhage is ineffective, which surgical procedure is appropriate to correct the cause of this condition? a. Hysterectomy b. Laparoscopy c. Laparotomy d. D&C

d. D&C

A primigravida at 39 weeks of gestation is observed for 2 hours in the intrapartum unit. The fetal heart rate has been normal. Contractions are 5 to 9 minutes apart, 20 to 30 seconds in duration, and of mild intensity. Cervical dilation is 1 to 2 cm and uneffaced (unchanged from admission). Membranes are intact. The nurse should expect the woman to be: a. Admitted and prepared for a cesarean birth. b. Admitted for extended observation. c. Discharged home with a sedative. d. Discharged home to await the onset of true labor.

d. Discharged home to await the onset of true labor.

A primigravida at 39 weeks of gestation is observed for 2 hours in the intrapartum unit. The fetal heart rate has been normal. Contractions are 5 to 9 minutes apart, 20 to 30 seconds in duration, and of mild intensity. Cervical dilation is 1 to 2 cm and uneffaced (unchanged from admission). Membranes are intact. The nurse should expect the woman to be: a. Admitted and prepared for a cesarean birth. b. Admitted for extended observation. c. Discharged home with a sedative. d. Discharged home to await the onset of true labor.

d. Discharged home to await the onset of true labor.

The nurse who performs vaginal examinations to assess a womans progress in labor should: a. Perform an examination at least once every hour during the active phase of labor. b. Perform the examination with the woman in the supine position. c. Wear two clean gloves for each examination. d. Discuss the findings with the woman and her partner.

d. Discuss the findings with the woman and her partner.

While evaluating an external monitor tracing of a woman in active labor whose labor is being induced, the nurse notes that the fetal heart rate (FHR) begins to decelerate at the onset of several contractions and returns to baseline before each contraction ends. The nurse should: a. Change the woman's position. b. Discontinue the oxytocin infusion. c. Insert an internal monitor. d. Document the finding in the client's record.

d. Document the finding in the client's record.

Maternity nurses often have to answer questions about the many, sometimes unusual ways people have tried to make the birthing experience more comfortable. For instance, nurses should be aware that: a. Music supplied by the support person has to be discouraged because it could disturb others or upset the hospital routine. b. Women in labor can benefit from sitting in a bathtub, but they must limit immersion to no longer than 15 minutes at a time. c. Effleurage is permissible, but counterpressure is almost always counterproductive. d. Electrodes attached to either side of the spine to provide high-intensity electrical impulses facilitate the release of endorphins.

d. Electrodes attached to either side of the spine to provide high-intensity electrical impulses facilitate the release of endorphins.

Increasing the infusion rate of nonadditive intravenous fluids can increase fetal oxygenation primarily by: a. Maintaining normal maternal temperature. b. Preventing normal maternal hypoglycemia. c. Increasing the oxygen-carrying capacity of the maternal blood. d. Expanding maternal blood volume.

d. Expanding maternal blood volume.

For the labor nurse, care of the expectant mother begins with any or all of these situations, with the exception of: a. The onset of progressive, regular contractions. b. The bloody, or pink, show. c. The spontaneous rupture of membranes. d. Formulation of the woman's plan of care for labor.

d. Formulation of the woman's plan of care for labor.

While completing a newborn assessment, the nurse should be aware that the most common birth injury is: a. To the soft tissues. b. Caused by forceps gripping the head on delivery. c. Fracture of the humerus and femur. d. Fracture of the clavicle.

d. Fracture of the clavicle.

All parents are entitled to a birthing environment in which breastfeeding is promoted and supported. The Baby Friendly Hospital Initiative endorsed by WHO and UNICEF was founded to encourage institutions to offer optimal levels of care for lactating mothers. Which instruction is not included in the Ten Steps to Successful Breastfeeding for Hospitals? a. Give newborns no food or drink other than breast milk. b. Have a written breastfeeding policy that is communicated to all staff. c. Help mothers initiate breastfeeding within one half hour of birth. d. Give artificial teats or pacifiers as necessary.

d. Give artificial teats or pacifiers as necessary.

The nurse caring for a family during a loss may notice that survival guilt sometimes is felt at the death of an infant by the child's: a. Siblings. b. Mother. c. Father. d. Grandparents.

d. Grandparents.

When assessing the preterm infant the nurse understands that compared with the term infant, the preterm infant has: a. Few blood vessels visible through the skin. b. More subcutaneous fat. c. Well-developed flexor muscles. d. Greater surface area in proportion to weight.

d. Greater surface area in proportion to weight.

What bacterial infection is definitely decreasing because of effective drug treatment? a. Escherichia coli infection b. Tuberculosis c. Candidiasis d. Group B streptococcal infection

d. Group B streptococcal infection

A postpartum woman telephones about her 4-day-old infant. She is not scheduled for a weight check until the infant is 10 days old, and she is worried about whether breastfeeding is going well. Effective breastfeeding is indicated by the newborn who: a. Sleeps for 6 hours at a time between feedings. b. Has at least one breast milk stool every 24 hours. c. Gains 1 to 2 ounces per week. d. Has at least six to eight wet diapers per day.

d. Has at least six to eight wet diapers per day.

In the current practice of childbirth preparation, emphasis is placed on: a. The Dick-Read (natural) childbirth method. b. The Lamaze (psychoprophylactic) method. c. The Bradley (husband-coached) method. d. Having expectant parents attend childbirth preparation in any or no specific method.

d. Having expectant parents attend childbirth preparation in any or no specific method.

Which condition, not uncommon in pregnancy, is likely to require careful medical assessment during the puerperium? a. Varicosities of the legs b. Carpal tunnel syndrome c. Periodic numbness and tingling of the fingers d. Headaches

d. Headaches

A woman in the active phase of the first stage of labor is using a shallow pattern of breathing, which is about twice the normal adult breathing rate. She starts to complain about feeling lightheaded and dizzy and states that her fingers are tingling. The nurse should: a. Notify the woman's physician. b. Tell the woman to slow the pace of her breathing. c. Administer oxygen via a mask or nasal cannula. d. Help her breathe into a paper bag

d. Help her breathe into a paper bag

As a perinatal nurse you realize that a fetal heart rate that is tachycardic, is bradycardic, or has late decelerations or loss of variability is nonreassuring and is associated with: a. Hypotension. b. Cord compression. c. Maternal drug use. d. Hypoxemia.

d. Hypoxemia.

After giving birth to a stillborn infant, the woman turns to the nurse and says, I just finished painting the baby's room. Do you think that caused my baby to die? The nurses best response to this woman is: a. That's an old wives tale; lots of women are around paint during pregnancy, and this doesn't happen to them. b. That's not likely. Paint is associated with elevated pediatric lead levels. c. Silence. d. I can understand your need to find an answer to what caused this. What else are you thinking about?

d. I can understand your need to find an answer to what caused this. What else are you thinking

Childbirth may result in injuries to the vagina and uterus. Pelvic floor exercises also known as Kegel exercises will help to strengthen the perineal muscles and encourage healing. The nurse knows that the client understands the correct process for completing these conditioning exercises when she reports: a. I contract my thighs, buttocks, and abdomen. b. I do 10 of these exercises every day. c. I stand while practicing this new exercise routine. d. I pretend that I am trying to stop the flow of urine midstream.

d. I pretend that I am trying to stop the flow of urine midstream.

A 25-year-old multiparous woman gave birth to an infant boy 1 day ago. Today her husband brings a large container of brown seaweed soup to the hospital. When the nurse enters the room, the husband asks for help with warming the soup so that his wife can eat it. The nurses most appropriate response is to ask the woman: a. Didn't you like your lunch? b. Does your doctor know that you are planning to eat that? c. What is that anyway? d. I'll warm the soup in the microwave for you.

d. I'll warm the soup in the microwave for you.

A new father states, I know nothing about babies, but he seems to be interested in learning. This is an ideal opportunity for the nurse to: a. Continue to observe his interaction with the newborn. b. Tell him when he does something wrong. c. Show no concern, as he will learn on his own. d. Include him in teaching sessions.

d. Include him in teaching sessions.

An Apgar score of 10 at 1 minute after birth would indicate a(n): a. Infant having no difficulty adjusting to extrauterine life and needing no further testing. b. Infant in severe distress who needs resuscitation. c. Prediction of a future free of neurologic problems. d. Infant having no difficulty to extrauterine life but who should be assessed again at 5 minutes after birth.

d. Infant having no difficulty to extrauterine life but who should be assessed again at 5 minutes after birth.

Because a full bladder prevents the uterus from contracting normally, nurses intervene to help the woman empty her bladder spontaneously as soon as possible. If all else fails, the last thing the nurse could try is: a. Pouring water from a squeeze bottle over the woman's perineum. b. Placing oil of peppermint in a bedpan under the woman. c. Asking the physician to prescribe analgesics. d. Inserting a sterile catheter.

d. Inserting a sterile catheter.

Nurses should be aware that the induction of labor: a. Can be achieved by external and internal version techniques. b. Is also known as a trial of labor (TOL). c. Is almost always done for medical reasons. d. Is rated for viability by a Bishop score.

d. Is rated for viability by a Bishop score.

Which statement describing the first phase of the transition period is inaccurate? a. It lasts no longer than 30 minutes. b. It is marked by spontaneous tremors, crying, and head movements. c. It includes the passage of meconium. d. It may involve the infants suddenly sleeping briefly.

d. It may involve the infants suddenly sleeping briefly.

Which fetal heart rate (FHR) finding would concern the nurse during labor? a. Accelerations with fetal movement b. Early decelerations c. An average FHR of 126 beats/mind. Late decelerations

d. Late decelerations

If a woman complains of back labor pain, the nurse could best suggest that she: a. Lie on her back for a while with her knees bent. b. Do less walking around. c. Take some deep, cleansing breaths. d. Lean over a birth ball with her knees on the floor.

d. Lean over a birth ball with her knees on the floor.

Which statement correctly describes the effects of various pain factors? a. Higher prostaglandin levels arising from dysmenorrhea can blunt the pain of childbirth. b. Upright positions in labor increase the pain factor because they cause greater fatigue. c. Women who move around trying different positions are experiencing more pain. d. Levels of pain-mitigating b-endorphins are higher during a spontaneous, natural childbirth.

d. Levels of pain-mitigating b-endorphins are higher during a spontaneous, natural childbirth.

A woman gave birth to a healthy infant boy 5 days ago. What type of lochia would the nurse expect to find when assessing this woman? a. Lochia rubra b. Lochia sangra c. Lochia alba d. Lochia serosa

d. Lochia serosa

Which maternal event is abnormal in the early postpartum period? a. Diuresis and diaphoresis b. Flatulence and constipation c. Extreme hunger and thirst d. Lochial color changes from rubra to alba

d. Lochial color changes from rubra to alba

To provide adequate postpartum care, the nurse should be aware that postpartum depression (PPD) with psychotic features: a. Is more likely to occur in women with more than two children. b. Is rarely delusional and then is usually about someone trying to harm her (the mother). c. Although serious, is not likely to need psychiatric hospitalization. d. May include bipolar disorder (formerly called manic depression).

d. May include bipolar disorder (formerly called manic depression).

An African-American woman noticed some bruises on her newborn girls buttocks. She asks the nurse who spanked her daughter. The nurse explains that these marks are called: a. Lanugo. b. Vascular nevi. c. Nevus flammeus. d. Mongolian spots.

d. Mongolian spots.

A macrosomic infant is born after a difficult forceps-assisted delivery. After stabilization the infant is weighed, and the birth weight is 4550 g (9 pounds, 6 ounces). The nurses most appropriate action is to: a. Leave the infant in the room with the mother. b. Take the infant immediately to the nursery. c. Perform a gestational age assessment to determine whether the infant is large for gestational age. d. Monitor blood glucose levels frequently and observe closely for signs of hypoglycemia.

d. Monitor blood glucose levels frequently and observe closely for signs of hypoglycemia.

While evaluating the reflexes of a newborn, the nurse notes that with a loud noise the newborn symmetrically abducts and extends his arms, his fingers fan out and form a C with the thumb and forefinger, and he has a slight tremor. The nurse would document this finding as a positive: a. Tonic neck reflex. b. Glabellar (Myerson) reflex. c. Babinski reflex. d. Moro reflex.

d. Moro reflex.

You are evaluating the fetal monitor tracing of your client, who is in active labor. Suddenly you see the fetal heart rate (FHR) drop from its baseline of 125 down to 80. You reposition the mother, provide oxygen, increase intravenous (IV) fluid, and perform a vaginal examination. The cervix has not changed. Five minutes have passed, and the fetal heart rate remains in the 80s. What additional nursing measures should you take? a. Scream for help. b. Insert a Foley catheter. c. Start Pitocin. d. Notify the care provider immediately.

d. Notify the care provider immediately.

In evaluating the effectiveness of magnesium sulfate for the treatment of preterm labor, what finding would alert the nurse to possible side effects? a. Urine output of 160 mL in 4 hours b. Deep tendon reflexes 2+ and no clonus c. Respiratory rate of 16 breaths/min d. Serum magnesium level of 10 mg/dL

d. Serum magnesium level of 10 mg/dL

Nursing care in the fourth trimester includes an important intervention sometimes referred to as taking the time to mother the mother. Specifically this expression refers to: a. Formally initializing individualized care by confirming the woman's and infants identification (ID) numbers on their respective wrist bands. (This is your baby.) b. Teaching the mother to check the identity of any person who comes to remove the baby from the room. (Its a dangerous world out there.) c. Including other family members in the teaching of self-care and child care. (Were all in this together.) d. Nurturing the woman by providing encouragement and support as she takes on the many tasks of motherhood.

d. Nurturing the woman by providing encouragement and support as she takes on the many tasks of motherhood.

For a woman at 42 weeks of gestation, which finding would require further assessment by the nurse? a. Fetal heart rate of 116 beats/min b. Cervix dilated 2 cm and 50% effaced c. Score of 8 on the biophysical profile d. One fetal movement noted in 1 hour of assessment by the mother

d. One fetal movement noted in 1 hour of assessment by the mother

As part of their teaching function at discharge, nurses should educate parents regarding safe sleep. Which statement is incorrect? a. Prevent exposure to people with upper respiratory tract infections. b. Keep the infant away from secondhand smoke. c. Avoid loose bedding, water beds, and beanbag chairs. d. Place the infant on his or her abdomen to sleep.

d. Place the infant on his or her abdomen to sleep.

Which basic type of pelvis includes the correct description and percentage of occurrence in women? a. Gynecoid: classic female; heart shaped; 75% b. Android: resembling the male; wider oval; 15% c. Anthropoid: resembling the ape; narrower; 10% d. Platypelloid: flattened, wide, shallow; 3%

d. Platypelloid: flattened, wide, shallow; 3%

Which assessment is least likely to be associated with a breech presentation? a. Meconium-stained amniotic fluid b. Fetal heart tones heard at or above the maternal umbilicus c. Preterm labor and birth d. Post-term gestation

d. Post-term gestation

After giving birth to a healthy infant boy, a primiparous woman, 16 years old, is admitted to the postpartum unit. An appropriate nursing diagnosis for her at this time is Risk for impaired parenting related to deficient knowledge of newborn care. In planning for the woman's discharge, what should the nurse be certain to include in the plan of care? a. Instruct the patient how to feed and bathe her infant. b. Give the patient written information on bathing her infant. c. Advise the patient that all mothers instinctively know how to care for their infants. d. Provide time for the patient to bathe her infant after she views an infant bath demonstration.

d. Provide time for the patient to bathe her infant after she views an infant bath

The interval between the birth of the newborn and the return of the reproductive organs to their normal nonpregnant state is called the: a. Involutionary period because of what happens to the uterus. b. Lochia period because of the nature of the vaginal discharge. c. Mini-tri period because it lasts only 3 to 6 weeks. d. Puerperium, or fourth trimester of pregnancy.

d. Puerperium, or fourth trimester of pregnancy.

In order to care for obstetric patients adequately, the nurse understands that labor contractions facilitate cervical dilation by: a. Contracting the lower uterine segment. b. Enlarging the internal size of the uterus. c. Promoting blood flow to the cervix. d. Pulling the cervix over the fetus and amniotic sac.

d. Pulling the cervix over the fetus and amniotic sac.

With shortened hospital stays, new mothers are often discharged before they begin to experience symptoms of the baby blues or postpartum depression. As part of the discharge teaching, the nurse can prepare the mother for this adjustment to her new role by instructing her regarding self-care activities to help prevent postpartum depression. The most accurate statement as related to these activities is to: a. Stay home and avoid outside activities to ensure adequate rest. b. Be certain that you are the only caregiver for your baby, to facilitate infant attachment. c. Keep feelings of sadness and adjustment to your new role to yourself. d. Realize that this is a common occurrence that affects many women.

d. Realize that this is a common occurrence that affects many women.

A nurse is observing a family. The mother is holding the baby she delivered less than 24 hours ago. Her husband is watching his wife and asking questions about newborn care. The 4-year-old brother is punching his mother on the back. The nurse should: a. Report the incident to the social services department. b. Advise the parents that the toddler needs to be reprimanded. c. Report to oncoming staff that the mother is probably not a good disciplinarian. d. Realize that this is a normal family adjusting to family change.

d. Realize that this is a normal family adjusting to family change.

An 18-year-old pregnant woman, gravida 1, is admitted to the labor and birth unit with moderate contractions every 5 minutes that last 40 seconds. The woman states, My contractions are so strong that I don't know what to do with myself. The nurse should: a. Assess for fetal well-being. b. Encourage the woman to lie on her side. c. Disturb the woman as little as possible. d. Recognize that pain is personalized for each individual.

d. Recognize that pain is personalized for each individual.

Because of the premature infants decreased immune functioning, what nursing diagnosis should the nurse include in a plan of care for a premature infant? a. Delayed growth and development b. Ineffective thermoregulation c. Ineffective infant feeding pattern d. Risk for infection

d. Risk for infection

The standard of care for obstetrics dictates that an internal version may be used to manipulate the: a. Fetus from a breech to a cephalic presentation before labor begins. b. Fetus from a transverse lie to a longitudinal lie before cesarean birth. c. Second twin from an oblique lie to a transverse lie before labor begins. d. Second twin from a transverse lie to a breech presentation during vaginal birth.

d. Second twin from a transverse lie to a breech presentation during vaginal birth.

A pregnant woman at 37 weeks of gestation has had ruptured membranes for 26 hours. A cesarean section is performed for failure to progress. The fetal heart rate (FHR) before birth is 180 beats/min with limited variability. At birth the newborn has Apgar scores of 6 and 7 at 1 and 5 minutes and is noted to be pale and tachypneic. On the basis of the maternal history, the cause of this newborns distress is most likely to be: a. Hypoglycemia. b. Phrenic nerve injury. c. Respiratory distress syndrome. d. Sepsis.

d. Sepsis.

Nurses should be able to teach breastfeeding mothers the signs that the infant has latched on correctly. Which statement indicates a poor latch? a. She feels a firm tugging sensation on her nipples but not pinching or pain. b. The baby sucks with cheeks rounded, not dimpled. c. The baby's jaw glides smoothly with sucking. d. She hears a clicking or smacking sound.

d. She hears a clicking or smacking sound.

Postbirth uterine/vaginal discharge, called lochia: a. Is similar to a light menstrual period for the first 6 to 12 hours. b. Is usually greater after cesarean births. c. Will usually decrease with ambulation and breastfeeding. d. Should smell like normal menstrual flow unless an infection is present.

d. Should smell like normal menstrual flow unless an infection is present.

One of the first symptoms of puerperal infection to assess for in the postpartum woman is: a. Fatigue continuing for longer than 1 week. b. Pain with voiding. c. Profuse vaginal bleeding with ambulation. d. Temperature of 38 C (100.4 F) or higher on 2 successive days starting 24 hours after birth.

d. Temperature of 38 C (100.4 F) or higher on 2 successive days starting 24 hours after birth.

A woman at 26 weeks of gestation is being assessed to determine whether she is experiencing preterm labor. What finding indicates that preterm labor is occurring? a. Estriol is not found in maternal saliva. b. Irregular, mild uterine contractions are occurring every 12 to 15 minutes. c. Fetal fibronectin is present in vaginal secretions. d. The cervix is effacing and dilated to 2 cm.

d. The cervix is effacing and dilated to 2 cm.

With regard to the care management of preterm labor, nurses should be aware that: a. Because all women must be considered at risk for preterm labor and prediction is so hit-and-miss, teaching pregnant women the symptoms probably causes more harm through false alarms. b. Braxton Hicks contractions often signal the onset of preterm labor. c. Because preterm labor is likely to be the start of an extended labor, a woman with symptoms can wait several hours before contacting the primary caregiver. d. The diagnosis of preterm labor is based on gestational age, uterine activity, and progressive cervical change.

d. The diagnosis of preterm labor is based on gestational age, uterine activity, and progressive cervical change.

In order to evaluate the condition of the patient accurately during labor, the nurse should be aware that: a. The woman's blood pressure will increase during contractions and fall back to prelabor normal between contractions. b. Use of the Valsalva maneuver is encouraged during the second stage of labor to relieve fetal hypoxia. c. Having the woman point her toes will reduce leg cramps. d. The endogenous endorphins released during labor will raise the woman's pain threshold and produce sedation.

d. The endogenous endorphins released during labor will raise the woman's pain threshold and produce sedation.

With regard to hemolytic diseases of the newborn, nurses should be aware that: a. Rh incompatibility matters only when an Rh-negative child is born to an Rh-positive mother. b. ABO incompatibility is more likely than Rh incompatibility to precipitate significant anemia. c. Exchange transfusions frequently are required in the treatment of hemolytic disorders. d. The indirect Coombs test is performed on the mother before birth; the direct Coombs test is performed on the cord blood after birth.

d. The indirect Coombs test is performed on the mother before birth; the direct Coombs test is performed on the cord blood after birth.

Early this morning, an infant boy was circumcised using the PlastiBell method. The nurse tells the mother that she and the infant can be discharged after: a. The bleeding stops completely. b. Yellow exudate forms over the glans. c. The PlastiBell rim falls off. d. The infant voids.

d. The infant voids.

When providing an infant with a gavage feeding, which of the following should be documented each time? a. The infants abdominal circumference after the feeding b. The infants heart rate and respirations c. The infants suck and swallow coordination d. The infants response to the feeding

d. The infants response to the feeding

A nursing student is helping the nursery nurses with morning vital signs. A baby born 10 hours ago by cesarean section is found to have moist lung sounds. What is the best interpretation of these data? a. The nurse should notify the pediatrician stat for this emergency situation. b. The neonate must have aspirated surfactant. c. If this baby was born vaginally, it could indicate a pneumothorax. d. The lungs of a baby delivered by cesarean section may sound moist for 24 hours after birth.

d. The lungs of a baby delivered by cesarean section may sound moist for 24 hours after birth.

A newly delivered mother who intends to breastfeed tells her nurse, I am so relieved that this pregnancy is over so I can start smoking again. The nurse encourages the client to refrain from smoking. However, this new mother insists that she will resume smoking. The nurse will need to adapt her health teaching to ensure that the client is aware that: a. Smoking has little or no effect on milk production. b. There is no relation between smoking and the time of feedings. c. The effects of secondhand smoke on infants are less significant than for adults. d. The mother should always smoke in another room.

d. The mother should always smoke in another room.

When responding to the question Will I produce enough milk for my baby as she grows and needs more milk at each feeding? the nurse should explain that: a. The breast milk will gradually become richer to supply additional calories. b. As the infant requires more milk, feedings can be supplemented with cows milk. c. Early addition of baby food will meet the infants needs. d. The mothers milk supply will increase as the infant demands more at each feeding.

d. The mothers milk supply will increase as the infant demands more at each feeding.

In a variation of rooming-in, called couplet care, the mother and infant share a room, and the mother shares the care of the infant with: a. The father of the infant. b. Her mother (the infants grandmother). c. Her eldest daughter (the infants sister). d. The nurse.

d. The nurse.

A woman gave birth vaginally to a 9-pound, 12-ounce girl yesterday. Her primary health care provider has written orders for perineal ice packs, use of a sitz bath tid, and a stool softener. What information is most closely correlated with these orders? a. The woman is a gravida 2, para 2. b. The woman had a vacuum-assisted birth. c. The woman received epidural anesthesia. d. The woman has an episiotomy.

d. The woman has an episiotomy.

A postpartum patient asks, Will these stretch marks go away? The nurses best response is: a. They will continue to fade and should be gone by your 6-week checkup. b. No, never. c. Yes, eventually. d. They will fade to silvery lines but won`t disappear completely.

d. They will fade to silvery lines but won`t disappear completely.

HIV may be perinatally transmitted: a. Only in the third trimester from the maternal circulation. b. From the use of unsterile instruments. c. Only through the ingestion of amniotic fluid. d. Through the ingestion of breast milk from an infected mother.

d. Through the ingestion of breast milk from an infected mother.

Human immunodeficiency virus (HIV) may be perinatally transmitted: a. Only in the third trimester from the maternal circulation. b. By a needlestick injury at birth from unsterile instruments. c. Only through the ingestion of amniotic fluid. d. Through the ingestion of breast milk from an infected mother.

d. Through the ingestion of breast milk from an infected mother.

A nurse may be called on to stimulate the fetal scalp: a. As part of fetal scalp blood sampling. b. In response to tocolysis. c. In preparation for fetal oxygen saturation monitoring. d. To elicit an acceleration in the fetal heart rate (FHR).

d. To elicit an acceleration in the fetal heart rate (FHR).

Nurses need to know the basic definitions and incidence data about postpartum hemorrhage (PPH). For instance: a. PPH is easy to recognize early; after all, the woman is bleeding. b. Traditionally it takes more than 1000 mL of blood after vaginal birth and 2500 mL after cesarean birth to define the condition as PPH. c. If anything, nurses and doctors tend to overestimate the amount of blood loss. d. Traditionally PPH has been classified as early or late with respect to birth.

d. Traditionally PPH has been classified as early or late with respect to birth.

On day 3 of life, a newborn continues to require 100% oxygen by nasal cannula. The parents ask whether they can hold their infant during his next gavage feeding. Given that this newborn is physiologically stable, what response would the nurse give? a. Parents are not allowed to hold infants who depend on oxygen. b. You may hold only your baby's hand during the feeding. c. Feedings cause more physiologic stress, so the baby must be closely monitored. Therefore, I don't think you should hold the baby. d. You may hold your baby during the feeding.

d. You may hold your baby during the feeding.


Conjuntos de estudio relacionados

U.S. ARMY Maps - Marginal Information and Map Symbols

View Set

AP Chemistry Chapter 12 Test Review

View Set

Jewish History - Terms and Concepts #1

View Set

LinkedIn Microsoft Word Skill Assessment

View Set